You are on page 1of 79

Question 1

The following statement regarding 1984 modification of Gustilo and Anderson's


classification of open fractures is true:
Type II injuries are heavily contaminated
Type IIIB injuries have adequate soft tissue cover
Type I injuries have a wound less than 2 cm long
Type IIIA injuries have adequate soft tissue cover despite extensive lacerations or flaps
All arterial injuries are graded Type IIIC regardless whether the arterial injury requires repair
or not.

References
Type III was reclassified into A,B and C in the 1984 modification. Gustilo, RB., Mendoza
RM., Williams DN. Problems in the management of type III open fractures: a new
classification. journal of trauma 24:742-746, 1984
J Orthop Trauma. 1993;7(4):357-60.
Interobserver reliability in the Gustilo and Anderson classification of open fractures.
Horn BD, Rettig ME
J Am Acad Orthop Surg. 2010 Jan;18(1):10-9.
Open tibial shaft fractures: I. Evaluation and initial wound management.
Melvin JS, Dombroski DG, Torbert JT, Kovach SJ, Esterhai JL, Mehta S.
Open fracture classification after Gustilo and Anderson
Grade I
Small in-to-out wound (< 1 cm). Low-energy injury, no soft-tissue crushing
or stripping, no bony comminution.
Grade II
Laceration over fracture (1-10 cm wound). Low-energy injury, minimal soft-tissue
crushing or stripping, no bony comminution.
Grade IIIA
Grade IIIB
Grade IIIC
High-energy injury, bony comminution or segmental fracture, soft-tissue crushing
stripping or serious contamination but adequate soft-tissue cover after d6bridement.
High-energy injury, bony comminution or segmental fracture, soft-tissue crushing
stripping or serious contamination, inadequate soft-tissue cover after d6bridement
Any open fracture associated with a local vascular injury requiring repair.
the severity of the wound to other clinicians and thus
prevent repeated and potentially contaminating
wound examinations outside of a sterile environment.

SURGICAL CLASSIFICATION
Formal classification of the open fracture occurs after
surgical debridement. The most widely used system is
that attributed to Gustilo and Anderson. 7,8 This classifies
the injury into three types, with the high-energy
injuries subdivided into three further groups (Table 2).
This system has stood the test of time and is widely
understood. The essential elements being the differentiation
between high- and low-energy injuries and the
concept of adequacy of soft-tissue cover in the highenergy
injuries.
The site of the fracture has an important effect on
the injury grade, particularly with regard to quantity
of local muscle cover and the vulnerability of the local
soft tissue to injury. A fracture in the normal femur
has to be high-energy because of the intrinsic strength
of the bone; accordingly all open femur fractures
must be grade III, although the thick muscle cover of
the thigh usually provides adequate soft-tissue cover.
In comparison, the tibia is unique with regard to its
poor soft-tissue cover and particular vulnerability to
injury. Accordingly, it is difficult to have a grade IIIa
injury to the tibial diaphysis where most high-energy
diaphyseal and distal tibial fractures are grade IIIb
and require complex reconstructive surgery. The most
severe injury (IIIc) is the open fracture associated with
a vascular injury that requires repair. These can occur
with either a severe or very a relatively 'minor' would.
The high grading is because of the primary risk to the
limb associated with the arterial disruption and the
susceptibility of ischaemic muscle to infection.
A similar classification system for closed fractures
has been proposed by Tscherne. 4 This also describes
the range of soft-tissue injuries from essentially no
significant injury to major vascular disruption, severe
muscle revitalization or compartment syndrome, all
within a closed skin envelope, a fuller classification
system considering both injury types is provided by
the comprehensive classification from the AO. 9 This
individually classifies the integument, nerve and vascular
tissues producing a more comprehensive and
detailed system

Question 2
This condition is caused by a point mutation for the gene encoding fibroblast growth
factor receptor 3 (FGFR3)?
Ehlers Danlos Syndrome
Marfan's Syndrome
Duchenne's Muscular Dystrophy

Achondroplasia
Becker Muscular Dystrophy

References
www.orthoteers.com
www.wheelessonline.com
Autosomal dominant disorder.
Arises due to a point mutation for the gene encoding a fibroblast growth factor (Fibroblast
growth factor receptor 3).
over 80% of all persons with this disorder are born to parents who are not achondroplastic, &
such parents rarely have second achondroplastic child.
It is presumed in these instances that disorder is consequence of mutation and may be
related to a mutation in a fibroblast growth factor gene.

Question 3
All the following with respect to the structure of the normal adult intervertebral disc
are correct EXCEPT:
The nucleus pulposus predominantly contains type II collagen
Nerve fibres are present on the inner and outer aspect of the annulus fibrosus
The intervertebral disc is relatively avascular in adults
The disc is sustained by diffusion and convection of nutrients
The annulus fibrosus has predominantly type I collagen

References
Basic Orthopaedic Science : The Stanmore Guide
Ramachandran et al
Hodder & Arnold
Histology and Pathology of the Human Intervertebral Disc
Roberts et al
JBJS(Am) 2006;88:10-14
Nerves and blood vessels are both present to a limited degree in the healthy adult disc,
restricted to the outer few millimeters of the anulus fibrosus. A small number of
mechanoreceptors are also present, most commonly having the morphology of Golgi tendon
organs, a few Ruffini receptors, and even fewer pacinian corpuscles.

Question 4
Which of the following describes the relationship between intensity of an x-ray beam
and the distance from its source?

Attenuation coefficient
Bragg's law
Compton effect
Inverse square law
Photoelectric effect

References
Manoj Ramachandran, The Stanmore guide: Pg 51
The intesity of an x-ray is indirectly related to the square of the distance from its source - the
inverse square law

Question 5
When performing a unilateral wrist arthrodesis, the following answer is TRUE:
15 degrees ulnar deviation is preferred
10-20 degrees dorsiflexion is ideal to preserve grip strength
Ulna-triquetral abutment cannot be avoided
An open epiphyseal plate in the distal radius is not a contraindication
The most common surgical approach is volar

References
Wheeless' Textbook of Orthopaedics
dorsal approach is common, an open epiphyseal plate is a contraindication, neutral to 5
degrees of ulna deviation is preferred, proximal row carpectomy overcomes ulna-triquetral
abutment

Question 6
Which nerve is at greatest risk during proximal locking during humeral nailing?
Radial Nerve
Median Nerve
Musculocutaneous Nerve
Axillary Nerve
Ulnar Nerve

References

Question 7
Which annular tendon pulleys are essential for normal flexor tendon function in the
hand?:
1 and 2
1 and 3
2 and 3
2 and 4
1 and 4

References
Campbell's

Operative

Orthopaedcis

"annular" pulleys 2 and 4 are essential to prevent bowstringing of the tendons and hence should
be preserved

Question 8
Which of the following is NOT a contraindication for Vertebroplasty?
Vertebral haemangioma
Spinal cord compression
Untreated coagulopathy
Vertebral fracture with posterior cortical defect
Vertebral osteomyelitis

References
Percutaneous Vertebroplasty for Pain Relief and Spinal Stabilization
Spine 2000: 25(8); 923-928
1) Untreated Coagulopathy - danger of causing intraspinal haematoma with subsequent cord
compression
2) Vertebral Fracture with posterior cortical defect - danger of cement exudating into the
canal causing compression
3) Vertebral Osteomyelitis - risk of disseminating infection via vertrebral vasculature
4) Cord Compression - a realtive contraindication (depends on where compression is in
relation to collapsed vertebra)
Haemangioma - filling these malformations with cement can often stop local expansion

Question 9
Which of the following anticoagulants potentiates the effect of Antithrombin III ?

Aspirin
Warfarin
Heparin
Clopidogrel
Dextran

References
Heparin forms a complex with antithrombin III, a protease inhibitor that inactivates thrombin. The
complex
inhibits
factor
Xa
only,
resulting
in
less
bleeding.
Basic Orthopaedic Science - The Stanmore Guide, Pg35

Question 10
The risk of osteonecrosis in a type II Hawkin's fracture is:
0-10%
10-20%
20-50%
50-80%
80-100%

References
Fortin PT, Balazsy JE: Talus fractures: Evaluation and treatment. J Am Orthop Surg 2001;9:
114 - 127
Type 1 is 0-13%
Type 2 is 20-50%
Type 3/4 is 80-100%

Question 11
Clotting studies on a patient with haemophilia A are most likely to show the following
Prolonged APTT and PT
Prolonged APTT and normal PT
Prolonged APTT and reduced PT
Normal APTT and prolonged PT
Reduced APTT and prolonged PT

References

Prothrombin time is measure of extrinsic pathway of coagulation and activated partial


thromboplastin time is measure of intrinsic pathway of coagulation. Heamophila A accounts
for 80% of all heamophilias and is due to factor VIII deficiency.
Prolonged APTT and PT: Factor V,X deficiency, DIC, Warfarin
Prolonged APTT and normal PT: Hemophilia, Von Willebrand's disease
Normal APTT and prolonged PT: early liver failure
www.gpnotebook.com
www.wheelesonline.com

Question 12
The nerve most at risk and during the Latarjet procedure is:
Axillary nerve
Musculocutaneous nerve
Brachial plexus lateral cord
Brachial plexus medial cord
Radial nerve

References
Campbell's Operative Orthopaedics
10th edition
Vol 3 - Sports Medicine
2412-5

Question 13
In Legg Calve Perthes disease, Group B patients (according to the Herring lateral
pillar classification) have a prognosis best described by which of the following
statements?
Uniformly good outcome in all age groups.
Uniformly poor outcome in all age groups.
Better than group A but worse than group C.
Worse outcome in patients with bone age more than 6 years at presentation.
Worse outcome in patients with bone age more than 4 years at presentation.

References
MD Miller, Review of orthopaedics. 4th Edn, 2004. Pediatric Orthopaedics: pp 145-193.
The outcome of Herring group B hips can vary with age. Overall children with bone age more
than 6 years at presentation do worse.

Group B hips do better than group C but worse than group A because the extent of
epiphyseal involvement is less than group C but more than group A.

Question 14
All the following mechanisms are important for haemostasis EXCEPT
Vascular spasm
Formation of platelet plug
Organization of blood clot
Fibrin degradation products
Calcium

References
Fibrin degradation product (FDPs), also known as fibrin split products, are components of
the blood produced by clot degeneration. These are produced by the action of plasmin on
deposited fibrin. The levels of these FDPs rises after any thrombotic event. It can be used to
test for disseminated intravascular coagulation.

Question 15
With regards to cam type lesion in Femoro-acetabular impingement, which of the
following is NOT true:
Caused by shear forces of the nonspherical portion of the femoral head against the
acetabulum.
Results in a characteristic pattern of anterosuperior cartilage loss over the femoral head and
corresponding dome
Predisposing factors that have been associated with femoroacetabular impingement include
increased femoral anteversion
Result of repetitive contact stresses of a normal femoral neck against an abnormal anterior
acetabular rim
Cannot be managed arthroscopically

References
Repetitive contact stresses of a normal femoral neck against an abnormal anterior
acetabular rim leads to 'pincer' type of femoroacetabular impingement
Shindle, Michael KM, Voos JamesE, Heyworth Benton E, Mintz Douglas N, Moya Luis E,
Buly Robert L, Kelly Bryan T. Hip Arthroscopy in the Athletic Patient: Current Techniques and
Spectrum of Disease.J Bone J Surg(A). 2007;89A(10):29-43

Question 16
The correct maximum dose of 0.5% bupivacaine (without adrenaline), that can be
used for local analgesia following a knee arthroscopy in an 80kg female patient is?
16ml
24ml
32ml
48ml
56ml

References
MRCS Core Modules: Essential Revision Notes 2nd edition. Sam Andrews. Pastest. p34
A 0.5% solution of bupivacaine contains 5mg/ml. The maximum safe concentration of
bupivacaine without adrenaline is 2mg/kg. Therefore the maximum safe dose in an 80kg
patient is 32ml (160mg).

Question 17
When selecting patients for 1st MTPJ cheilectomy, which factor has been shown to
correlate with a poor outcome
Onychogryphosis
Bilateral disease
>50% articular cartilage loss
25% articular cartilage loss
Previous hip or knee replacement surgery

References
Factors associated with poor clinical outcome with chielectomy are global pain, pain on the
plantar aspect, severely decreased range of movement, severe degerative changes on Xray
and > 50% articular cartilage involvement on intraoperative examination of joint.
1. Coughlin MJ, Shurnas PS.Hallux rigidus.J Bone Joint Surg Am. 2004 Sep;86-A Suppl 1(Pt
2):119-30
2. Cheilectomy: still a useful technique for grade I and grade II hallux limitus/rigidus.
Geldwert JJ, Rock GD, McGrath MP, Mancuso JE.J Foot Surg. 1992 Mar-Apr;31(2):154-9.
3. Surgery of the Foot & Ankle : 8Th Edition; Michael J Coughlin, Roger A. Mann, Charles L.
Saltzman; Chapter 16;Pg880

Question 18
Which of the following does NOT lead to a thrombotic tendency?
Factor V Leiden
Antithrombin deficiency
Protein C and S deficiency
Heparininduced thrombocytopenia
Anti-Protein Z deficiency

References
Anesth Analg. 2010 Nov 16.
Etiology and Assessment of Hypercoagulability with Lessons from Heparin-Induced
Thrombocytopenia.
Sniecinski RM, Hursting MJ, Paidas MJ, Levy JH.
Thromb Res. 2008;121(6):727-34.
Protein Z levels and anti-protein Z antibodies in patients with arterial and venous thrombosis.
Pardos-Gea J, Ordi-Ros J, Serrano S, Balada E, Nicolau I, Vilardell M.

Question 19
Which is the gold standard test for diagnosing post-operative Deep Vein Thrmobosis
(DVT)?
Duplex Ultrasonography
125 I labelled fibrinogen
D-Dimer
Venography
Impedance Plethysmography

References
Duplex: good for proximal thrombi but poor for calf or pelvis, cheap and non invasive
therefore most common
Venography: gold standard, used in most clinical trials but costly and invasive
D Dimer: very non specific, costly and high in postoperative setting
AAOS Comprehensive orthopaedic review page 155

Question 20
The Ideberg classification is useful when planning surgical treatment of glenoid
fractures. A horizontal fracture through the glenoid and scapular body with no
additional glenoid fracture comprises an Ideberg type:

I
II
III
IV
V

References
Ideberg R. Fractures of the scapula involving the glenoid fossa. In:
Bateman JE, Welsh RP, eds. Surgery of the shoulder. Philadelphia:Decker, 1984:63-6.
Wheeless online textbook

Question 21
3rd generation of cementing includes all of the following EXCEPT
Distal femoral plug
Proximal finger pressurisation for femur
Vacuum mixing
Pulse lavage
Femoral stem centraliser

References
The differing generations of cementing techniques are described below : First
Hand mix Rasp only, leave cancellous bone. Manual insertion with finger packing
Second Hand mix More aggressive rasp, brushing pulsatile lavage. Cement gun
Distal canal plug. Early distal centralizers
Third Vacuum mix / centrifugation More aggressive rasp, brushing pulsatile lavage. Cement
gun with pressurization. Distal canal plug Proximal and distal centralizers
Mulroy RD Jr, Harris WH. The effect of improved cementing techniques on component
loosening in total hip replacement. An 11-year radiographic review. J Bone Joint Surg Br.
1990; 72:757-760.
Miller 4th Edition - Page 268

Question 22
Which of the following statements regarding diathermy is FALSE?
Temperatures of 1000 degrees centigrade can be reached
When monopolar diathermy is used the patient plate should cover an area of atleast 70cm2

Bipolar diathermy is generally less powerful than monopolar and does not require a patient
plate electrode
Bipolar diathermy can be used in both the "coagulation" and "cutting" modes
Bipolar diathermy is generally prefered over monopolar, if the patient has a cardiac
pacemaker.

References
Diathermy: passes high frequency alternating current through body
Neuromuscular stimulation disappears above 50KHz
Monopolar: current from generator to small active electrode, tip represents high current
density, flow then occurs through patient and to plate
Bipolar: current passes from one limb of forceps to another. Only tissue between tips is
heated
Essential surgical practice: Cuschieri

Question 23
The typical anterior approach to the cervical spine uses the plane between which of
the following structures?
The carotid artery (laterally) and jugular vein (medially)
The carotid sheath (laterally) and the trachea and esophagus (medially)
The inferior thyroid artery (medially) and the carotid sheath (laterally)
The brachial plexus (laterally) and the carotid sheath (medially)
The trachea (laterally) and the esophagus (medially)

References
Hoppenfeld & DeBoer - Surgical Exposures in Orthopaedics: The Anatomic Approach
(Publishers - LWW)
There are 3 layers of fascia which govern the anterior approach to the neck. The first is the
investing layer of deep fascia (the most superficial) which envelopes sternocleidomastoid
and surrounds the neck like a collar. Incise this to retract SCM laterally. After you go through
this then in order to retract the carotid sheath laterally you need to incise the pretracheal
fascia which is continous with the carotid sheath. Finally the prevertebral fascia overlies the
cervical spine.

Question 24
The chemical mediator responsible for osteolysis seen with failing total hip
replacement is

TGF B
IL-1
IL-4
Leukotrienes
IL-10

References
The Basic Science of Periprosthetic Osteolysis. Michael J. Archibeck, Joshua J. Jacobs,
Kenneth A. Roebuck and Tibor T. Glant. J Bone Joint Surg Am. 2000;82:1478.
IL-1, IL-6, TNF-a, and PGE2 are believed to be among the most important components
capable of inducing cell proliferation, promoting osteoclast formation, and stimulating
osteoclasts to resorb the adjacent bone

Question 25
Which one of the following statements is FALSE about trigger thumbs in childhood:
Under 9 months of age 30% resolve spontaneously
Over the age of 1 year, less than 10% resolve spontaneously.
30% are bilateral.
Surgical release of the A1 pulley is the treatment of choice in children over the age of 1 year.
There is a strong familial inheritance pattern.

References
There is no familial inheritance pattern. Rest of the statements are correct.
Trigger digits on the other hand are seen with neurologic syndromes (Trisomy 18) and
mucopolysaccharidoses.
Transverse incision in the digital crease is to be preferred for a better scar.
Lovell and Winter's Pediatric Orthopaedics, 6th Edn 2006. The Upper Limb, pp 922-985.

Question 26
Amongst the following, which has the greatest range of axial rotation?
Upper Thoracic spine
Thoracolumbar spine
Lower Thoracic spine
Lumbosacral spine

Equal in all of the above

References
White AA, Panjabi MM. (1990). Clinical biomechanics of the spine. Philadelphia, JB
Lippincott.
Axial rotation is 8 from T1-T8. This is largely due to the coronal orientation of the facets in
the thoracic spine which allow rotatory movement to occur. The axial rotation of the lower
thoracic spine and thoracolumbar junction is reduced to 2 below T10 due to the transition to
more sagittally oriented facets.

Question 27
Which of the following statements is FALSE?
About 15% of all bone tumours are primary spine tumours
Combined surgical decompression and radiotherapy is generally superior to radiotherapy
alone in the treatment of metastatic spinal cord compression
Spine metastases from renal cell carcinoma tend to be hypervascular
Spinal tumours often present with pain and weakness
Ependymoma is the most common type of intramedullary tumor in adults

References
Patchell R, Tibbs PA, Regine F, et al: A randomized trial of direct decompressive surgical
resection in the treatment of spinal cord compression caused by metastasis. Lancet
366:643-648, 2005
1) Less than 5% of all bone tumours are primary spine tumours
2) Radiotherapy and cord decompression is superior to radiotherapy alone
3) Symptoms are variable with pain and weakness being the commonest presenting feature
4) Ependymomas are the most common in adults with astrocytomas being more common in
children
5) Renal cell carcinoma metastases bleed a lot and embolisation prior to surgery is helpful

Question 28
The rigidity or bending stiffness of a plate is proportional to
The thickness of the plate
The thickness of the plate to the second power
The thickness of the plate to the third power
The length of the plate
The number of screws

References
The bending stiffness is proportional to the second moment area, for a rectangle this is
proportional to the distance from the neutral axis to the power of three.
Therefore doubling the plate thickness increases its bending stiffness eightfold.
The torsional stiffness in a cylinder is proportional to the effective radius of the object to the
power 4.
Review of Orthopaedics, Miller
Ramachandran Basic Sciences

Question 29
Typical theatre operating light illumination is
40000 lux
4 lux
4 mega lux
4 giga lux
400 lux

References
Light intensity at the surgical site should be 40000 lux. Satellite lamps are associated with
turbulent airflow.
Ramachandran Basic Sciences

Question 30
In the process of wound healing, macrophages
Precede the appearance of neutrophils
Are activated by interleukin-2
Synthesise and secrete tumour necrosis factor
Are derived from megakaryocytes
Are not involved in humoral immunity

References
There are four phases to wound healing. Haemostasis, inflammation, proliferation and
remodelling
Platelets aggregate to allow haemostasis and are fragments of megakaryocytes
Neutrophils predominate initally followed by fibroblasts and macrophages (at 2 days)
Macrophages are involved in humoral and cell mediated immunity and secrete growth
factors and cytokines
IL-2 stimulates T cell proliferation

Question 31
15 year old presents with back pain. Plain radiographs show vertebra plana at L1.
Which is the most likely diagnosis?
Discitis
Eosinophillic granuloma
Osteoblastoma
Scheurmanns
Rickets

References
Floman Y, Bar-On E, Moshieff R, Mirovsky Y, Robin GC, Ramu N. Eosiniophillic Granuloma
of the Spine. J Pediatr Orthop B 1997 Oct;6(4) 260-5

Question 32
Which of the following is NOT a common cause for spasmodic torticollis.
Sternocleidomastoid trauma during delivery
Peritonsillar abscess
Retropharyngeal abscess
Spinal haematoma
Cervical trauma

References
http://emedicine.medscape.com/article/794191-overview - good summary
Sternocleidomastoid trauma at birth is thought to be the mechanism for congenital muscular
torticollis

Question 33
A 30 year old woman presents to your clinic with pain in the groin. Your examination
findings are consistent with a diagnosis of hip impingement. Which investigation is
most suitable for excluding a labral tear?
Computed tomography with 3D reconstruction
Single photo-emission computed tomography
Plain X-ray arthrogram

MRI arthrogram with gadolinium contrast


Triple phase bone scan

References
Hip Arthoscopy Richard N. Villar
MRI Arthrogram of the hip is most suitable investigation

Question 34
Which structure is at most risk of damage in performing trans-articular C1/C2 screw
fixation
C2 nerve root
Hypoglossal nerve
Carotid artery
Vertebral artery
Vagus nerve

References
J.P.Stannard. Surgical treatment of orthopaedic trauma. 2007, pg122-123
Anterolateral C1-C2 Transarticular Fixation for Atlantoaxial Arthrodesis: Landmarks, Working
Area, and Angles of Approach.
Cavalcanti et al
Neurosurgery 2010 Sept 67(3) p.38-42 (this paper has a great picture)
The vertebral artery adopts a serpentine course in relationship to the C2 vertebra, making it
susceptible to injury during the surgical procedures in the region. The multiple loops of the
artery and a buffer space within the vertebral artery groove on the inferior surface of the
superior facet of the C2 vertebra and over the posterior arch of atlas provide the artery an
extra length and space, probably essential to avoid any stretch during neck movements.

Question 35
The most common method of performing a z-plasty in hand surgery theoretically yields
75% of lengthening along the line of the central limb. Which method should be used to
achieve this?
45-degree angle between the limbs with one limb 10% longer than the other
45-degree angle between the limbs of equal length
60-degree angle between the limbs with one limb 10% longer than the other
60-degree angle between the limbs of equal length
90-degree angle between the limbs of equal length

References
Green DP, Hotchkiss RN, Pederson C, Wolfe SW, eds: Green's Operative Hand urgery, 5th Ed,
p1655.
Philadelphia,
Churchill
Livingstone,
2005.
A z-plasty is a form of transposition flap. While the lengths of the limbs should always be equal,
the angles may vary to change the amount of desired lengthening. The commonly used 60degree angle theoretically yields 75% lengthening along the line of the central limb.

Question 36
What type of afferent peripheral nerve fibre is responsible for transmitting vibration
sense to the spinal cord
A (A alpha)
A (A beta)
A (A gamma)
A (A delta)
C

References
Miller, MD; Hart, JA: Review of orthopaedics, 5th Edition, Saunders 2008.
Cutaneous and subcutaneous mechanoreceptors called Ruffinis corpuscles convey
vibration sense via the A beta afferent nerve fibre to the spinal cord via dorsal root ganglion.
A beta also transmits light touch(Meissner's corpuscles), pressure(Merkel's receptor) and
flutter(Pacini's corpuscle).
A delta transmits sharp, pricking pain detected by the peripheral nociceptors.
A alpha transmits limb proprioception via the golgi tendon apparatus and muscle spindle
primary mechanoreceptors.
A gamma transmits sharp pain due to thermal injury.
Type C transmits slow burning pain via nociceptors.
A beta also mediates limb proprioception via the muscle spindle secondary and joint capsule
mechanoreceptors.

Question 37
At what temperature must an amputated finger be transferred for re-implantation
10 degrees centigrade
0 degrees centigrade
4 degrees centigrade
Room temperature
Body temperature

References
Campbell

Operative

Orthopaedics

13th

edition

The amputated part must be placed in a plastic bag and sealed and transported in a container
with ice

Question 38
In Dupuytren's disease, the spiral cord receives a contribution from which ligament?
Cleland's ligament
Grayson's ligament
Natatory ligament
Distal commisural ligament
Sagittal fibres

References
Greens Operative Hand Surgery, Chapter 5
The spiral cord arises from the longitudinal pretendinous fibers and follows layer 2 through
to the lateral digital sheet. From there it gains attachment to the middle phalanx by
Grayson's ligament.

Question 39
Which of the following material has a modulus of elasticity closest to that of cortical
bone (0.7 - 4.9 GPa)?
Aluminium hydroxide
Cobalt-chromium-molybdenum
Polymethylmethacrylate
Stainless steel
Titanium

References
1. Review of Orthopaedics, 5th Edition, Mark D. Miller (editor), Saunders, an imprint of
Elsevier, Philadelphia, Copyright 2008
2. Buckwalter JA, Einhorn RA, Simon SR (eds): Orthopaedic Basic Science: Biology and
Biomechanics of the Musculoskeletal System, Rosemont IL, American Academy of
Orthopaedic Surgeons, 2000.

Question 40
The nerve supplying teres minor is a branch of which nerve?
Suprascapular nerve (C5,C6)
Lower subscapular nerve (C5,C6)
Upper subscapular nerves (C5,C6)
Axillary nerve (C5,C6)
Medial supraclavicular nerves (C3,C4)

References
Sinnatamby C.S. Last's Anatomy, Regional and Applied. Eleventh edition. 2006, pg 48
Review of orthopaedics, MD Miller, 4th Edition pg 600
The nerve supplying teres minor originates from the posterior branch of axillary nerve

Question 41
Which of the following would be an evidence based indication for operative intervention
for a typical thoraco-lumbar burst fracture of the spine?
Canal encroachment without neurological deficit
Kyphosis greater than 30 degrees
Canal encroachment with static neurological deficit
Loss of anterior vertebral body height of less than 50%
Lack of posterior midline tenderness on examination

References
Canal encroachment and neurological deficit are not indications per se although progressive
neurological deficit would be. Other relative indications include injury to the posterior longitudinal
ligament
complex.
Gertzbein did the classic large study which gave us the concept that a kyphosis of >30 degrees
correlates
with
persistent
pain
Therapeutic
Decision
Making
in
Thoracolumbar
Spine
Trauma
Cumher
et
al
Spine
2010
Volume
35
Issue
21S
Gertzbein SD (1992) Scoliosis Research Society multicenter spine fracture study. Spine 17:525
540

Question 42
Regarding obtaining consent for an operative intervention, which of the following
statements is true?
The refusal of a competent person aged 16-17 can never be over-ridden by either a person with
parental responsibility or a court.

Refusal by a competent child and all persons with parental responsibility for the child can be
over-ruled by the court if the welfare of the child so requires.
A parent or relative can give consent on behalf of an adult unable to give consent for him or
herself (an incapable adult).
When a patient gives valid consent to an intervention, in general that consent remains valid for a
period of four weeks after which time it must be re-obtained.
Consent may only be obtained by the clinician providing the treatment or investigation

References
Reference
guide
to
consent
for
examination
or
treatment,
March
http://www.gosh.nhs.uk/clinical_information/clinical_guidelines/cpg_guideline_00151

2001

Question 43
According to the Musculoskeletal Tumour Society, excision of a tumour through the
pseudo-capsule of a tumour (the reactive zone) is termed
Amputation
Intra-lesional
Marginal
Radical
Wide

References
Sim FH, Frassica DA: Soft tissue tumours: Diagnosis, evaluation and management. AAOS 2:209,
1994

Question 44
When considering sagittal plane balancing in Total Knee Replacement, down sizing of the
femoral component (Decreasing the AP dimension) will help in balancing the knee in
which of the following situations?
Tight extension gap, normal flexion gap
Tight extension gap, tight flexion gap
Normal extension gap, tight flexion gap
Loose extension gap, loose flexion gap
Normal extension gap, loose flexion gap

References
Miller

MD.

Review

of

Orthopaedics

4th

ed

pp287.

Posterior femoral cut and the proximal tibial cut mark the upper and lower limits of the flexion

space. Therefore downsizing the femoral component will increase the flexion gap without altering
the extension gap. This will be a suitable option when the flexion gap is tight and the extension
gap is ok.

Question 45
Which of the following makes up part of the posterior column of the spine, according to
Denis?
Posterior annulus
Posterior vertebral body
Posterior ligamentous complex
Posterior longitudinal ligament
Transverse processes

References
Denis F. Spinal instability as defined by the three-column concept in acute spinal trauma. Clin
Orthop
Rel
Res
Oct
1984;(189):65-76
The anterior column consists of: anterior 2/3 of vertebtral body, anterior annulus, anterior
longitudinal
ligament.
Middle column: posterior wall of vertebral body, posterior annulus, posterior longitudinal ligament
Posterior column: Posterior ligamentous complex connecting neural arches - consisting of facet
capsules,
ligamentum
flavum,
interspinous
ligament
&
supraspinous
ligament.
Failure

of

two

or

more

columns

generally

results

in

instability.

Don't get confused between the posterior longitudinal ligamentous complex and the posterior
spinal ligamentous complex.

Question 46
Stabilisation methods for management of hand injuries use the anatomical
consideration that MCP joint is stable on flexion and lax in extension. This is because
of the following reason:
Eccentric origin of the collateral ligament on the metacarpal head
Eccentric attachment of the lumbricals to the metacarpal
Eccentric attachment of extensor tendon to the metacarpal
Eccentric origin of inter-osseous muscles to the metacarpal
Central attachment of the lumbricals to the metacarpal

References
Text book: Skeletal Trauma - Volume Two. 3rd Edn. Browner, Jupiter, Levine, Trafton. page
1154-1155 (chapter 38: Fractures and Dislocations of the hand)

The collateral ligaments of the MCP joint originate dorsal to the axis of flexion and insert
broadly along the palmar aspects of the proximal phalanges. It is partly because of this
eccentric origin of the collateral ligaments on the oddly shaped metacarpal head that the
MCP joint is more lax in extension and more stable in flexion.

Question 47
Which of the following conditions will not affect key grip in the hand?
Rheumatoid arthritis
Carpal tunnel syndrome
Injury to the recurrent branch of the median nerve
Ulnar nerve injury at Guyon's canal
Radial nerve injury

References
In rheumatoid arthritis the MCP jt radial collateral ligament is attenuated and weak on pinch.
In carpal tunnel syndrome, median nerve involvement can affect the thenar musculature.
In ulnar nerve injury at Guyon's canal, the adductor pollicis muscle is affected.

Question 48
A 40 year old computer programmer presented with a painless focal swelling on the palm
of his hand. He takes phenytoin for his well-controlled epilepsy and occasionally requires
salbutamol inhaler for his asthma. The most likely cause of his focal hand swelling is:
Rheumatoid arthritis
Dupuytrens contracture
Implantation dermoid
Suppurative tenosynovitis
Scleroderma

References
Greens

Operative

Hand

Surgery.

Chapter

A link
to
diabetes
is
accepted,
but
this
type
of
Dupuytren's
disease
has
a
slightly
different
form,
seemingly
more
widespread,
especially
on
the
radial
digits,
and
often
less
progressive.
Alcohol
remains
a
controversial
association,
as
does
tobacco
consumption.
The
increased
incidence in patients with epilepsy may be due to the drugs rather than to the disease.

Question 49

Which of the following statements concerning posterior shoulder dislocation is true?


The internal rotators are not as powerful as the external rotators
The inferior glenohumeral ligament will invariably be normal
Activities of daily living are affected in chronic cases
Reverse Hill Sachs is a risk factor for recurrence
After closed reduction, the position of external rotation should be avoided.

References
JJP Warner, Complex and revision problems in Shoulder Surgery, 2nd Edition chapter 3
lat dorsi and subscap much more powerful than infraspinatus and teres minor. Inferior gleno
humeral ligament, ant and post portions will be damaged in posterior dislocation, Schwartz,
Warren and O Brien, Posterior shoulder instability, Orthop Clin North Am 1987;18:409.
ADLS are not affected in almost all cases of posterior instability, Hawkins et al jbjs 1984;66A:169.
Reverse Hill Sachs defect contributes to ongoing posterior dislocation/subluxation Pollock +
Bigliani,
Clin
Orthop
1993;291:85.
After closed reduction external rotation may be a necessary position to prevent redislocation.

Question 50
A patient with far lateral disc prolapse at L4/L5 is likely to present with:
Loss of ankle reflex.
Altered sensation on the dorsum of the foot in the first web space
Weakness of Extensor Hallucis Longus
Foot drop
Weakness of Flexor Hallucis Longus

References
Miller

D:

Review

of

Orthopaedics.

Sauders,

2006.

Far lateral disc prolapse at L4/5 affects the exiting L4 nerve root. A paracentral prolapse would
affect
the
traversing
L5
root.
Ankle
jerk
S1
1st
web
space
of
the
foot
innervation
L5
Extensor
hallucis
longus
L5
Flexor Hallucis longus - S1

Question 51
Which one of following conditions is most suitable for ankle arthroplasty?
Ankle arthritis secondary to neuropathic joint disease
Ankle arthritis associated with severe mal-alignment of the ankle joint
Failed ankle arthrodesis
Rheumatoid ankle arthritis

Ankle arthritis associated with osteonecrosis of the talus

References
Surgery of the Foot & Ankle : 8Th Edition; Michael J Coughlin, Roger A. Mann, Charles L.
Saltzman;
Chapter
17
All choices except R.A ankle arthritis are indications of ankle arthrodesis over replacement

Question 52
Meyers & McKeever type III fractures of the tibial spine are best managed by Arthroscopic / open reduction and internal fixation.
Closed reduction under anaesthesia and casting.
Non weight bearing with crutches and regular observation.
Partial weight bearing with crutches and regular observation.
Aspiration of knee and casting.

References
Tachdjian's Pediatric Orthopaedics. 4th Edition, 2008. Lower Extremity Injuries; Knee: pp 26532712.
Type III fractures are best treated operatively with an arthroscopic or open technique using
suture or a screw (the choice depending upon the surgeon's expertise). The knee is held in full
extension for 4 weeks, followed by strengthening and range of motion exercises.
The inter-meniscal ligament keeps the fragment from sitting down in its bed and needs to be
fished
out
of
the
fracture.
If untreated or inadequately treated, type III fractures can result in ACL deficiency symptoms and
signs and can act as a mechanical block to full extension.

Question 53
In which of the following scenarios would thoracic back pain NOT be a common finding?
Thoracic spinal stenosis
Thoracic compression fracture
Thoracic discitis
Adolescent idiopathic thoracic scoliosis
Thoracic costovertebral arthritis

References
Sales et al 'Osteoarthritis of the costovertebral joints' JBJS (Br) 2007;89-B:1336-9
IDIOPATHIC adolescent scoliosis by and large does not cause back pain. In fact presence of

back pain in scoliosis should prompt further investigation eg MRI to exclude an underlying
disorder before being labelled as idiopathic.

Question 54
Non-operative treatment of distal biceps tendon rupture results in
Elbow flexion is weaker than supination
Supination is weaker than elbow flexion
Flexion strength is half that of supination
Equal weakness in both flexion and supination
No weakness in either flexion or supination

References
Rupture of the distal biceps tendon: biomechanical assessment of different treatment options.
WH
Norman.
CORR.
Vol
193.
1985.
p
189.
Miller,
Review
of
Orthopaedics,
5th
edition
Distal biceps tendon rupture occurs after forceful, eccentric overload of the partially flexed elbow.
Norman's paper reviews the outcome of 16 cases of distal biceps tendon rupture, finding
supination to be weaker than flexion. Repair can take place via a two incision or one incision
technique.

Question 55
Which of the following is NOT a recognised technique for evaluating postero-lateral
corner injuries?
Increased external rotation
Postero-lateral drawer test
Reverse pivot shift test
Dial test at 15 and 60
External rotation recurvatum test

References
Sekiya & Miller et al. A clinically relevant assessment of posterior cruciate ligament and
posterolateral corner injuries. J Bone Joint Surgery (Am) 2008; 90: 1621-7.
MD Miller. Review of Orthopaedics. Fourth edition. 2004. Saunders Elsevier. Page218
Posterolateral corner injuries are rarely isolated injuries and are commonly associated with
other ligament injuries, especially the PCL. Examination for increased external rotation,
external rotation recurvatum test and, posterolateral drawer test and reverse pivot shift test
are important. The Dial test should be performed at prone/supine (dial test) 30/90 degrees
knee flexion, with ER torque and 10 degree difference compared to contralateral leg is a
positive test.

Question 56
Local anaesthetic agents are less effective in the presence of infection due to which one
of the following options?
A higher ionised concentration of the agent
A lower ionised concentration of the agent
An alkaline pH in the tissue fluid
Local vasodilatation
Micro-organisms ingesting the local anaesthetic agent

References
Only the non-ionised form of local anaesthetics (LA) can traverse the neuronal membrane, in
order to access the sodium channels from inside the cell. The acidic environment, caused by the
presence of infection, results in a higher concentration of ionised LA. This produces relative
inactivity of the LA.

Question 57
The following are distal realignment procedures of the patella EXCEPT
Elmslie-Trillat procedure
Maquet procedure
Insall procedure
Fulkerson osteotomy
Hauser procedure

References
www.wheelessonline.com/ortho/distal_realignment_for_patellar_subluxation
http://www.wheelessonline.com/ortho/insall_proximal_realignment_for_disorders_of_the_patella
Fulkerson
Osteotomy:
- modification of the Elmslie-Trillat Procedure, but involves anterior displacement as well;
- main indications are persistent pain and moderate articular degeneration;
- allows anteriorization of upto 15 mm, which should decrease lateral facet contact pressure;
Hauser
Procedure:
discussed
for
historical
purposes
only;
- involves medialization of the tibial tubercle inorder to decrease Q angle;
- due to the anatomy of the proximal tibia, translating the tibial tubercle medially, will also
translate the tubercle posteriorly; posterior translation of the tibial tubercle will have the effect of
increasing
patellofemoral
contact
pressures
which
leads
to
DJD;
Maquet
Procedure:
- discussed for historical purposes only; involves anterior translation of the tibial tubercle which
has the effect of decreasing patellofemoral contact forces; patients w/ pain due to early
patellofemoral arthrosis may expect pain relief following the Maquet Procedure; -disadvantages

w/ this procedure include high incidence of skin necrosis, and no effect on the Q angle;
Elmslie-Trillat
Procedure:
medial
tibial
tubercle
transfer
which
has
no
posterior
displacement;
does
not
involve
anterior
displacement
of
the
tuberosity;
Insall procedure: Proximal realignment with standard lateral retinacular release.

Question 58
Osteochondritis dissecans most commonly affects which area of the knee?
Medial side of lateral femoral condyle
Lateral side of medial femoral condyle
Medial side of medial femoral condyle
Trochlea groove
Lateral side of lateral femoral condyle

References
Miller Review of Orthopaedics 5th edition
The most common area of involvement in the knee is the lateral side of the medial femoral
condyle. The lateral femoral condyle is involved in 15% of cases. It rarely affects the patella.
Resolution is more likely to occur in juvenile cases, and least likely in adults.

Question 59
The most common method of sterilising allograft tissue used in Orthopaedics is?
Moist heat.
Gamma irradiation.
Dry heat.
Ultraviolet light.
Ethylene oxide.

References
McAllister DR, Joyce MJ, Mann BJ, Vangsness CT Jr.
Allograft update: the current status of tissue regulation, procurement, processing, and
sterilization.
Am J Sports Med. 2007 Dec;35(12):2148-58. Epub 2007 Nov 1.
Ethylene oxide use has been found to increase risk of graft-host reactions and its use is now
less common. Gamma irradiation is effective but may be detrimental to the mechanical
properties of the tissue.

Question 60

Which of the following tests has been shown to be the most specific at testing for anterior
cruciate ligaments tears?
Trendelenburg
Pivot shift
Lachmann
McMurray's
Anterior draw

References
Clinical diagnosis of
Benjaminse
A
J
Orthop
Sports

an anterior cruciate ligament rupture:


Gokeler
A,
van
der
Phys
Ther
2006
May;
36

a meta-analysis.
Schans
CP
(5):
267-88

The anterior cruciate ligament is one of the most commonly injured structures in the knee. In a
meta-analysis of 28 studies, it was found that the pivot-shift test was the most specific test (98%),
although sensitivity was poor (24%). The Lachmann had a pooled sensitivity of 85%, and a
pooled specificity of 94%. The anterior draw test had a good sensitivity of 92% and specificity of
91% when used in chronic conditions, although was not as good in acute conditions. McMurray's
test is used to determine the presence of meniscal tears. The Trendelenburg test is used during
examination of the hip.

Question 61
Undisplaced fractures of the distal radius are associated with rupture of which
structure
Flexor Pollicis Longus
Extensor Digitorum
Extensor Pollicis Longus
Extensor Indicis Proprius
Flexor Pollicis Brevis

References
Helal B, Chen SC, Iwegbu G: Rupture of the extensor pollicis longus tendon in undisplaced
Colles type of fracture. Hand 1982;14:41-47

Question 62
Which nerve is at risk during dorsal approach of the wrist ?
Median nerve
Superficial radial nerve
Dorsal cutaneous branch of the ulnar nerve

Ulnar nerve
Dorsal cutaneous branch of the median nerve

References
Hoppenfeld 3rd Edition
Superficial radial nerve emerges from beneath the tendon of brachioradialis muscle just
above the wrist joint before travelling to the dorsum of the hand. Damage to nerves can
occur if dissection is begun within the fat. If the incision is taken down to the extensor
retinaculum before the radial flap is elevated, the nerve is protected

Question 63
59 year old man has increased pain in his right foot with deformity for 3 years.
Examination reveals hindfoot fixed valgus of 15 degrees with midfoot abduction. He
has tried nonsurgical treatment without relief. The most appropriate surgical
management would be
Medial sliding os calcis osteotomy
Subtalar arthrodesis
Talonavicular arthrodesis
Subtalar arthrodesis with FDL transfer
Triple arthrodesis

References
Core Knowledge in Orthopaedics. Foot and Ankle.
A fixed planovalgus foot is a Johnson and Strom grade 3 deformity which requires triple
arthrodesis

Question 64
A patient, who underwent in-situ pinning for SUFE and was asymptomatic following
surgery, develops pain, stiffness and flexion deformity. Most likely diagnosis is
AVN
Chondrolysis
Further slip
Normal
Pin penetration

References

Although rare, chondrolysis is one of the recognised complication of SUFE. It usually


presents as pain and significant restriction of the joint movement.
Tachdjian: Pediatric orthopaedics Second Edition Vol 2, Page no 1066

Question 65
The most common group of bacteria causing prosthetic joint infection is
Gram negative cocci
Gram positive bacilli
Gram positive cocci (coagulase negative)
Gram negative bacilli
Gram positive cocci (catalase negative)

References
Unusual Aerobic and Anaerobic Bacteria Associated with Prosthetic Joint Infections
Marculescu, Camelia E; Berbari, Elie F; Cockerill, Franklin R; Osmon, Douglas R
Clinical Orthopaedics and Related Research. 451, October 2006, 55-63
S. epidermidis, a coagulase-negative staphylococcus species, is a commensal of the skin
which frequently causes prosthetic joint infections.

Question 66
A displaced fracture of the talar neck with dislocation of the body of the talus (ankle
joint) but NOT the head (talonavicular joint) would be classified as:
Hawkins I
Hawkins II
Hawkins III
Hawkins IV
Hawkins V

References
Koval KJ, Zuckerman JD. Handbook of fractures. Chapter 40 p.437
Hawkins classification is important as it correlates well with the risk of subsequent talar
avascular necrosis.

Question 67
Regarding subtalar dislocation

Lateral dislocation is the more common


Medial dislocation requires open reduction more frequently than lateral dislocation
Successful closed reduction of lateral dislocation can be prevented by interposed posterior
tibial tendon.
Approximately 10% of lateral dislocations require open reduction
Is usually unstable once reduced

References
http://www.wheelessonline.com/ortho/sub_talar_dislocation
Lateral dislocation is a less common type of subtalar dislocation (15%). It occurs when
calcaneus is displaced lateral to talus; Talar head lies medially, and foot appears pronated;
Navicular lies lateral to the talar neck; lateral dislocations may be complicated by interposed
posterior tibial tendon (or sometimes FDL); approx 20% of lateral dislocations require open
reduction; incision over sinus tarsi, and three wks of NWB casting, followed by ROM;
Medial dislocation, is the most common sub talar dislocation (85%); foot & calcaneus are
displaced medially; head of the talus prominent dorsolaterally; navicular lies medial and
sometimes dorsal to talar head & neck; foot is plantar flexed and is supinated; inversion
causes this injury; approx of 10% of medial dislocations require open reduction; capsule of
talonaviclar joint & EDB blocks reduction, or in some cases the talar head may button hole
thru the EDB; medial sub talar dislocations are treated by longitudinal anteromedial incision
over prominent head and neck of talus & minipulation and release of interposed tissues;
since the joint is stable after reduction there is no need for internal fixation; after reduction, a
short leg cast is applied for 3 to 4 wks;

Question 68
Which major structure is at risk when retractors are placed behind the transverse
acetabular ligament during total hip replacement.
Pudendal nerve
Inferior gluteal nerve
Sciatic nerve
Obturator nerve
Inferior gluteal nerve and vessels

References
Millers

review

of

orthopaedics

pelvis

and

hip

anatomy.

Retractors placed behind the transverse acetabular ligament can injure the obturator nerve and
artery.

Question 69

Which of the hindfoot joints has the highest incidence of non-union following a triple
arthrodesis?
Calcaneocuboid
Talonavicular
Subtalar
Ankle
Calcaneonavicular

References
Mizel MS, Sobel M: Disorders of the Foot and Ankle. In Miller MD (ed): Review of Orthopaedics,
2nd
Ed.
Philadelphia.
W.B.
Saunders,
1996,
pp223-243.
Isolated talonavicular fusion is associated with a high non-union rate. This is thought to be
because its movement is closely linked to that of the subtalar joint. If the subtalar joint remains
mobile an excess load is placed through the talonavicular joint. Furthermore the navicular has a
poor blood supply.

Question 70
In addition to the acromion, which structure is considered to be involved in shoulder
impingement and should be released at the time of surgery
Superior glenohumeral ligament
Coracoacromial ligament
Middle glenohumeral ligament
Deltoid fascia
Coracoclavicular ligament

References
Failure to resect the anterior lateral bundle of the CAL can result in ongoing impingement.
The coracoacromial ligament: Morphology and study of acromial enthesopathy
Stephen Fealy, Ernest W. April, Michael Khazzam, Juan Armengol-Barallat, Louis U. Bigliani
Journal of Shoulder and Elbow Surgery
September 2005 (Vol. 14, Issue 5, Pages 542-548)
Fealy et al, JSES, 2006 (an abstract)

Question 71
Arthrodesis of the trapeziometacarpal joint of the thumb would be contraindicated in
which one of the followoing situations?
A young, active, manual worker
Stage 4 basal thumb arthritis

Subluxation of the thumb metacarpal


A posttraumatic cause
Adducted thumb metacarpal

References
33. Forseth M, Stern P. Complications of trapeziometacarpal arthrodesis using plate and
screw fixation. J Hand Surgery 2003; 28A :342-5.
Trapeziectomy is a good operation for pain relief but often at the expense of grip strength.
This may not be well tolerated in young active manual workers who may be better treated
with arthrodesis. Trapeziometacarpal arthrodesis is contraindicated if there is pantrapezial
arthritis (Eaton Littler Stage 4)

Question 72
Regarding rotator cuff tendinopathy of the shoulder
Radiographs often show migration of the humeral head inferiorly
Arthropathy is a relative contraindication to cuff repair
Articular tears are associated with a hook-shaped (type III) acromion
Most rotator cuff tears require surgery
Glenohumeral instability is a cause of primary impingement syndrome

References
Brinker
Miller
OKUP

Question 73
At which site does the medial collateral ligament of the knee most commonly rupture?
Femoral insertion
Mid-substance
Tibial insertion
Tibial Tuberosity
Pes anserinus

References

Rockwood & Green's: Fracture in Adult - Volume 2. 5th edition. 2005.


Answer - At the femoral insertion. MCL injury occurs as a result of valgus stressing of the
knee. Results are good for isolated MCL injury treated with a knee brace.

Question 74
Which of the following is true regarding rotator cuff tears?
Rotator cuff tears are almost always traumatic in origin
Rotator cuff tears are associated with overhead activity and osteoarthritis
Tears in the supraspinatus tendon rarely occur close to its insertion
Articular sided tears are 2-3 times more common than bursal tears
Bursal tears never heal

References
Rotator cuff tears can be traumatic in origin but are most commonly due chronic repetitive insults
such
as
subacromial
impingement
and
tendinopathy.
Tears are associated with increasing age, repetitive over head activity and inflammatory arthritis.
Tears most commonly occur in the supraspinatus tendon close to its insertion in the greater
tuberosity
(an
area
of
relative
hypovascularity)
Articular sided tears are 2-3 times more common than bursal sided tears. Bursal sided tears can
sometimes heal but articular sided tears rarely heal, possibly due to the inhibitory factors in the
synovial
fluid
Postgraduate orthopaedics. The candidates guide to the FRCS (Tr and Orth.)Examination

Question 75
In Developmental Dysplasia of the Hip (DDH), which of the following statements is
true regarding the Ortolani's test?
It is a provocative test for DDH that attempts to subluxate the femoral head.
It is often positive in the walking child.
It consists of pushing down the greater trochanter while simultaneously adducting the hip.
It consists of lifting up the greater trochanter while simultaneously abducting the hip.
It is always positive in the infant with DDH.

References
Tachdjian's Pediatric Orthopaedics. 4th Edition. 2008. Developmental Dysplasia of the Hip:
pp 637-770.
Barlow's and Ortolani's tests are used to diagnose DDH in neonates and infants.

Barlow's is the provocative test for the dislocatable hip and Ortolani is the relocating test for
the already 'out' hip. (Remember Ortolani for Out !)
The 'clunk' of Ortolani is a sensation of relocation of the head that is seen and/or felt but not
heard. Nevertheless the 'clicky' baby hip always needs to be further investigated via
ultrasonography.

Question 76
What is the best treatment for a localised superficial ulcer under the first metatarsal
head in a diabetic patient?
Compression graded bandaging
Total contact cast
Alginate dressing
Bed rest and elevation
Non-adhesive simple dressing

References
1. Myerson, Papa J, Eaton K, Wilson K. The total-contact cast for management of
neuropathic plantar ulceration of the foot. J.Bone Joint Surg.Am. 1992; 74:261-269
2. Greer Richardson E. Disorders of the foot and ankle. In: Miller M D, ed., Review of
Orthopaedics. Philadelphia, Saunders, 2004: 309-357
Total contact casting is the treatment of choice for Grade I (Localised superficial ulcer) and
Grade II (Deep ulcer to tendon, bone, ligament or joint) on the Wagner classification. A
formal debridement is generally needed before the application of the cast.

Question 77
Concerning periprosthetic fractures after shoulder arthroplasty, which of the
following is correct?
The incidence is reported as between 1% and 3%.
All type B fractures should be treated surgically
Non-operative management of type C fractures is associated with a poor outcome.
Well fixed type A fractures require long stem revision and bone grafting.
When revising to a long stem, the tip should extend at least 1 cortical diameter past the
fracture.

References
Kumar et al.Periprosthetic Humeral Fractures After Shoulder Arthroplasty. JBJS (Am) 86 (4):
680.2004.

Question 78
Which type of meniscal tear would be most suitable for repair?
Radial
Vertical
Horizontal
Degenerate
Oblique

References
J Am Acad Orthop Surg 2002;10:168-176

Question 79
Which nerve supplies the inferior gemellus?
Inferior gluteal nerve
Nerve to obturator internus
Nerve to quadratus femoris
Obturator nerve
Superior gluteal nerve

References
Whittaker RH and Borley NR. Instant Anatomy. Blackwell Science
Moore KL and Dalley AF. Clinically oriented anatomy. Lippincott, Williams and Wilkins.
Nerve to quadratus femoris supplies quadratus femoris and the inferior gemellus, as well as
a sensory branch to the hip joint. The superior gemellus is supplied via the nerve to obturator
internus. The inferior gluteal nerve supplies gluteus medius and minimus, while the superior
gluteal nerve supplies gluteus maximus. The obturator nerve supplies obturator externus,
the thigh adductors and sensory branches to hip, knee and skin of the medial thigh.

Question 80
Which of the following statements regarding the Latarjet procedure is true?
For best results the transferred coracoid process should be more than 5 mm medial to the
glenoid rim
Only the short head of biceps is transferred along with the coracoid
The fixation screw must not purchase the posterior glenoid cortex
The Latarjet procedure helps to restore a static stabiliser

Positioning the coracoid inferior to the transverse equator of the glenoid correlates to a better
outcome

References
Campbell's
Operative
Orthopaedic
This procedure is mainly performed when there is some bone loss from the front of the glenoid
(as a result of a bony bankart lesion or repeated dislocations wearing away the front of the
glenoid).
The coracoid is transferred, together with the coracobrachialis and the short head of thebiceps
tendons, to the 1/3 antero-inferior border of the glenoid, to obtain a dynamic reinforcement of the
inferiorpart of the capsule when the arm is abducted and externallyrotated. BristowLatarjet
procedure is a non-anatomic shoulder reconstruction procedure and it might modify the biomechanics of the shoulder

Question 81
An adductor tenotomy is often performed to improve the following abnormal gait pattern:
Vaulting gait
Trendelenburg gait
High-stepping gait
Scissoring gait
Ataxic gait

References
Adductor tenotomy is commonly performed in the child with CP who can stand with support but
has difficulty walking due to scissoring. Origins of adductor longus, gracilis and part of adductor
brevis
are
released.
Adductor
Magnus
is
not
released.
Differential
Diagnosis
of
a
Non-antalgic
Limp
in
a
child:
Equinus
Gait
(Toe-Walking):
Idiopathic tight Achilles tendon, Clubfoot (residual or untreated), Limb-length discrepancy,
Cerebral
palsy
Trendelenburg
Gait:
Legg-Calv-Perthes disease, Developmental dysplasia of the hip, Slipped capital femoral
epiphysis,
Muscular
dystrophy,
Hemiplegic
cerebral
palsy.
Circumduction
Gait/Vaulting
Gait:
Limb-length discrepancy, Cerebral palsy, Any cause of ankle or knee stiffness
Steppage
(High-stepping)
Gait:
Myelodysplasia, Cerebral palsy, Charcot-Marie-Tooth disease, Friedreichs ataxia Flynn JM,
Widmann RF, MD. The Limping Child: Evaluation and Diagnosis. J Am Acad Orthop Surg, Vol 9,
No
2,
March/April
2001,
89-98.
Tachdjian's Pediatric Orthopaedics. 4th Edition. 2008. Disorders of the Brain: pp 1275-1404.

Question 82
Which of the following statements is true regarding lengthening of the gastrosoleus
complex:

May result in a crouch gait in hemiplegic CP


It should be avoided in the ambulatory child
Lengthening at the level of the Tendo Achilles is contraindicated in cerebral palsy
It is often performed during correction of the cavovarus foot
Should not be performed by complete division of the Tendo Achilles

References
Ref: Campbells 11th edition, chapter 30 FOOT
Calf lengthening is performed for many different conditions. A complete percutaneous
Achilles tenotomy is required in the majority of cases of clubfoot correction by the Ponseti
method. Although aponeurotic lengthening is preferred in the diplegic form of CP in order to
avoid crouch gait, this risk is not associated with the hemiplegic form, so a TA lengthening is
a valid treatment option. A calf lengthening procedure is often required with correction of
cavovarus

Question 83
A 42 yr old man is undergoing a total knee replacement. He had previous recent
trauma to his knee with a longitudinal incision from previous surgery. If a different
skin incision is to be made what is the recommended distance (in cms) between the
incisions.
1
5
7
9
Does not matter.

References
A seven centimeter skin bridge represents the minimum safe distance between surgical
incisions.
Relevant surgical exposures
By Bernard F. Morrey, Matthew C. Morrey
Chapter 9 The Knee

Question 84
In a patient with established radial nerve palsy, which of the following tendon
transfers best addresses lack of wrist extension?
Pronator Teres to ECRL
Pronator Teres to ECRB
FCU to ERCB

Brachioradialis transfer to ERCB


Brachioradialis transfer to ERCL

References
In radial nerve palsy recognised transfers are:
wrist extension - PT to ECRB, Finger extension - FCU/FCR to EDC or FDS (Ring ) to EDC,
thumb - PL to EPL or FDS (Middle) to EPL
Hand Surgery
Richard A. Berger
www.Orthoteers.com
www.wheelessonline.com

Question 85
Which of the following is the primary antagonist to the Tibialis Anterior tendon?
Peroneus Brevis
Peroneus Longus
Flexor Hallucis Longus
Tibialis Posterior
Flexor Digitorum Longus

References
Surgery of the Foot & Ankle : 8Th Edition; Michael J Coughlin, Roger A. Mann, Charles L.
Saltzman; Chapter 21:page 1132
Peroneus longus is responsible for plantar flexion of First ray in Pes Cavus and is the
primary antagonist of tibialis anterior.
Tibialis posterior is the antagonist for the Peroneus brevis and is responsible for the
adduction of foot and inversion of the heel and subtalar joint in pes cavus.
Two main muscles involved in Pes Cavus secondary to CMTD are Peroneus brevis and
Tibialis anterior.

Question 86
The second web space on the dorsum of the foot is supplied by
Deep peroneal nerve
Common peroneal nerve
Superficial peroneal nerve
Sural nerve
Saphenous nerve

References
Last's Anatomy. Regional and Applied. Tenth edition. CS Sinnatamby.
The saphenous nerve supplies the medial border of the foot including the medial border of
great toe. The sural nerve supplies the lateral border of the foot. The deep peroneal nerve
supplies the first web space on the dorsum. Rest of the dorsum of the foot is supplied by the
superficial peroneal nerve.

Question 87
A 12 year old boy walks into the accident & emergency department with a one month
long history of right knee pain. Radiographs show a slipped capital femoral epiphysis
(Southwick angle 20 degrees). Which of the following is true?
There is at least a 50% chance that this child will develop a slip on the contralateral side
Safe surgical dislocation and subcapital realignment osteotomy is a reasonable treatment
option
When pinning in situ, screw entry should be proximal to the lesser trochanter
Ultrasound of the hip should be considered
This child has a high risk of developing avascular necrosis

References
Uglow, Clarke JBJS (Br) 2004; 86-B; 631-5.
Loder et al JBJS [Am] 1993;75-A:1134-40.
Southwick JBJS [Am] 1967;49-A:807-35.
The chances of a slip on the contralateral side are between 20 and 30%. Surgical dislocation
and realignment osteotomy is increasing in popularity for severe slips, but would not be
appropriate for a mild slip. It is best to avoid the lateral cortex with screw placement through
the anterior cortex to avoid the risk of fracture. Ultrasound would not contribute to
management in this case. It is a stable slip and so has a low risk of AVN

Question 88
With regards to the anterior approach (Smith-Peterson) to the hip, which of the
following statements is true?
The superficial dissection is between gracilis and sartorius
The internervous plane is between the obturator nerve and the femoral nerve
The medial femoral circumflex artery is a common structure at risk
The internervous plane is between the femoral nerve and the superior gluteal nerve
The superficial dissection is between rectus femoris and sartorius

References

This approach takes advantage of the interneural plane between the sartorius (femoral
nerve) and tensor fasciae latae (superior gluteal nerve). It is useful for operative procedures
such as open reduction of the congenitally dislocated hip. The lateral femoral cutaneous
nerve is retracted anteriorly, and the ascending branch of the lateral femoral circumflex
artery (which lies superficial to the rectus) is ligated. For deeper dissection, approach the
interval between the gluteus medius and rectus femoris. Detach the origin of both heads of
the rectus femoris. Reflection of the conjoined rectus tendon too distally can risk injury to the
descending branch of the lateral femoral circumflex artery. Retract the rectus medially and
the gluteus medius laterally. Dissect any attachments of the iliopsoas to the inferior capsule
and perform a capsulotomy. There is a risk to the lateral femoral cutaneous nerve, which is
located anterior or medial to the sartorius about 6-8 cm below the ASIS. The superficial
circumflex artery penetrates the tensor fasciae latae just anterior to the lateral femoral
cutaneous nerve. The femoral nerve and vessels can sometimes be injured by aggressive
medial retraction of the sartorius.
Millers review of Orthopaedics, 5th ed., pp176-7

Question 89
The use of a vascularised graft, in the treatment of AVN of the femoral head is
contraindicated in which of the following young patients?
Those in pain
Where the AVN involves the whole femoral head
Those with a small area of subchondral collapse
If the patient is still on steroids
Those with a low Ficat grade of AVN

References
Vascularized fibular strut graftingThis procedure represents a relatively new concept in treating
ON, which has been championed most recently by Urbanik. What is heard most about the
procedure is the core decompression and insertion of the vascularized fibular strut graft.
Vascularized fibular strut grafting is usually recommended for earlier stages of ON, but it has
been shown to be effective even if some subchondral collapse has occurred. Sometimes the
subchondral bone can even be tapped up to a more congruent position.
Vascularized fibular strut grafting is contraindicated when there is whole head involvement.
Pain, low ficat grading (grades I,II and early III - small subchondral collapse) are good
indications. Steroids are a common cause of AVN in the UK but are not stopped. Osteonecrosis Miller, 5th edition, p354

Question 90
Which of the following does NOT have an aetiological association with non-traumatic
osteonecrosis of the femoral head?
Corticosteroids
Excessive alcohol consumption
Human Immunodeficiency Virus (HIV) infection

Paracetamol overdose
Sickle cell disease

References
Osteonecrosis (AVN) of the hip has numerous causes. The commonest in the UK are
Idiopathic, alcoholism and steroids. Idiopathic ON may have an association with
hypercoagulability disorders.
Jay R. Lieberman, Daniel J. Berry, Michael A. Montv, Roy K. Aaron, John J. Callaghan, Amar
Rayadhyaksha, and James R. Urbaniak
Osteonecrosis of the Hip: Management in the Twenty-first Century
J. Bone Joint Surg. Am., May 2002; 84: 834 - 853.
Michael A. Mont, Lynne C. Jones, and David S. Hungerford
Nontraumatic Osteonecrosis of the Femoral Head: Ten Years Later
J. Bone Joint Surg. Am., May 2006; 88: 1117 - 1132.

Question 91
The following is INCORRECT regarding valgus proximal femoral osteotomy
The limb is inevitably lengthened
At least 15 degree of pre-operative abduction is necessary
Indicated for post Perthe's disease deformity
Involves medially based closing wedge
Abductor lever arm lengthened

References
This is a common operation for post Perthe's deformity. Miscellaneous Problems around the
hip, Oxford Textbook of Orthopaedics and Trauma, Oxford University Press, Vol 1: Pages
991-994
One residual of Legg-Calv-Perthes disease is a malformed femoral head with resulting
hinged abduction. Hinged abduction of the hip is an abnormal movement that occurs when
the deformed femoral head fails to slide within the acetabulum. A trench is formed laterally,
adjacent to a large uncovered portion of the deformed head anterolaterally. Campbells
operative orthopaedics, Vol. II, Part IX.
Raney et al. described valgus subtrochanteric osteotomy for malformed femoral heads with
hinge abduction. All were Catterall III and IV with previous failed treatment. At 5-year followup, 62% had satisfactory results. - The role of proximal femoral valgus osteotomy in LeggCalv-Perthes disease. Raney EM, Grogan DP, Hurley ME, Ogden MJ. Orthopedics. 2002
May;25(5):513-7.

Question 92
To obtain good patellar tracking during total knee replacement, a surgeon should:
Raise the joint line

Place the tibial component in slight internal rotation


Place the femoral component in slight external rotation
Position the patellar implant slightly laterally on the patella
Place the femoral component in a medialised position

References
The main reason for slight external rotation of the femoral component is to allow for the
change made to the axis of the joint line in TKR. The tibial bone cut has changed the angle
of the joint line from a natural 3 degrees of varus to a neutral horizontal axis, therefore in
order to achieve a equal flexion gap the femoral component must be externally rotated.
However, this external rotation of the femoral component also improves the tracking of the
patella.
The most common complications in TKR involve abnormal patellar tracking. Avoiding patellar
problems in knee replacement is essentially an exercise in Q angle management and
understanding the technical aspects that alter the knee joint line. All of the other responses
will increase the Q angle.
Millers review of orthopaedics, 5th ed. p330

Question 93
A 59 year old lady presents with a 3 month history of progressively increasing pain
around her right hip such that she now has difficulty weight bearing. Radiographs
show a lytic lesion in the proximal third of the femur involving 50% of the cortex. She
has no significant past medical history. An isotope bone scan performed shows a
single hot spot in the right subtrochanteric region. The next step in management of
this lady is:
CT scan of the chest, abdomen and pelvis
Needle biopsy of the lesion by an orthopaedic tumour surgeon
Radiotherapy to alleviate the pain
Surgical stabilisation followed by radiotherapy
Urgent surgical stabilisation using a reconstruction nail

References
The next appropriate step may not be the only appropriate step in the next stage of
management and therefore, the best choice has to be selected from the stems provided. For
example, an alternative correct answer (not provided in the choices) would be an MRI scan
of the lesion.
1. Oxford Textbook of Orthopaedics and Trauma Volume 1 p 340-347
2. Review of Orthopaedics, Miller 4th Ed. P 498-500
3. Hage D et al., Incidence, Location and Diagnostic Evaluation of Metastatic Bone Disease.
Orthopaedic Clinics of North America. Vol 31.4, Oct 2000, p 515-528
4. Ward W et al., Metastatic Disease of the Femur. Orthopaedic Clinics of North America. Vol
31.4, Oct 2000, p 633-645

5. Rougraff B, Indications for Operative Treatment. Orthopaedic Clinics of North America. Vol
31.4, Oct 2000, p 567-575

Question 94
When considering the hindfoot in Rheumatoid arthritis, which of the following is true?
The hindfoot deformity should be corrected before addressing angular deformities at the hip or
knee
The hindfoot most often develops a valgus deformity due to disruption of the talo-calcaneal
interosseous ligament
The hindfoot most often develops a valgus deformity due to shortening of the tendo-achilles
The hindfoot most often develops a varus deformity due to the driving forces from a hallux valgus
The hindfoot most often develops a varus deformity due to shortening of the tendo-achilles

References
The hindfoot in rheumatoid arthritis most often developes a valgus deformity due to disruption of
the talocalcaneal interosseous ligament. The ligament has a synovial covering which is
weakened by the rheumatoid process. This ligament is an important stabiliser of the subtalar
joint. Weight bearing forces the calcaneous into valgus and the subtalar joint, followed by the
midtarsal joint subluxates resulting in the typical deformities of hindfoot valgus, midfoot-forefoot
abduction and collapse of the medial longitudinal arch. Chronic hindfoot valgus results in
shortening
of
the
Achilles
tendon.
Angular deformities at the hip and knee should be corrected before realignment of the hindfoot or
ankle.
Miller, Review of Orthopaedics.

Question 95
Which of the following statements are true about adductor tenotomy in a child with
cerebral palsy?
Usually involves division of adductor longus
Usually involves division of adductor magnus
Usually involves division of adductor brevis
Should include a neurectomy of the anterior branch of the obturator nerve
Is a common cause of hip subluxation

References
Adductor myotomy is one of the oldest and most commonly performed operations in children
with CP. It usually involves gracilis and adductor longus. In severe deformities, the surgeon
may choose to divide the brevis and rarely, if ever, adductor magnus. Anterior branch
obturator neurectomy is usually avoided.
Morrissy and Weinstein. Atlas of Paediatric Orthopaedic Surgery, p379

Question 96
At what age does the proximal femoral epiphysis usually ossify?
In utero
1-3 months
4-6 months
6-12 months
After 12 months

References
Miller
The femoral epiphysis usually ossifies between 4 and 6 months of age; this can be delayed
in cases of DDH, both as a result of the condition, or as a consequence of the treatment, in
which case this most likely represents AVN.

Question 97
In the context of tarsal tunnel syndrome, how many branches of the tibial nerve are
present in the tarsal tunnel and should be protected during surgical release?
Two
Three
Four
Five
One

References
Campbells Operative Orthopaedics
The (posterior) tibial nerve divides into three branches within the tarsal tunnel. Two plantar
nerves (medial and lateral) and one calcaneal branch should be seen and protected during
surgical decompression.

Question 98
Which one of the following criteria is NOT included in Mirel's predictive scoring system
for impending pathological fracture
Degree of pain
Site of the lesion
Location of lesion in bone (metaphyseal, diaphyseal)
Size of lesion
Type of lesion

References
Mirel's scoring system incorporates the site of the lesion, pain, type of lesion and size.
Mirel H (1989). Metastatic disease in long bones. Clin Orthop 249; 256-264

Question 99
During the standard anterior approach for ankle arthroplasty, which structure is typically
NOT at risk?
Superficial peroneal nerve
Anterior tibial artery
Extensor hallucis longus tendon
Saphenous nerve
Deep peroneal nerve

References
Surgical Exposures in Orthopaedics, The Anatomical Approach. Hoppenfeld and deBoer

Question 100
With regards to the gait cycle, double-limb stance consists of approximately what
total percentage of one cycle (stride)
10%
12%
16%
20%
25%

References
One gait cycle (stride) is defined from initial contact (heel strike) of one heel until the next
heel strike of the ipsilateral heel. It can be divided into a stance phase and a swing phase. At
the beginning and end of the stance phase there is a period of double-limb support, each
consisting of approximately 10% of one total cycle. (2 x 10% = 20%) Miller Review of
Orthopaedics 5th edition, p553.

Question 101
A patient complains of pain and rubbing of his shoe on the top of his second toe.
Examination reveals a corn on the dorsum of the PIPJ with fixed flexion deformity of
the same joint. The DIPJ is flexible. The condition is:
Mallet toe
Hammer toe

Claw toe
Turf toe
Curly toe

References
Miller
Wheeless' textbook of Orthopaedics

Question 102
In Charcot-Marie-Tooth disease, the characteristic clinical foot deformity is:
1. Hindfoot varus / midfoot abduction / forefoot pronation (valgus)
2. Hindfoot varus / midfoot adduction / forefoot pronation (valgus)
3. Hindfoot varus / midfoot abduction / forefoot supination (varus)
4. Hindfoot valgus / midfoot adduction / forefoot supination (varus)
5. Hindfoot valgus / midfoot abduction / forefoot pronation (valgus)

References
Miller - Review of Orthopaedics
The characteristic deformity in Charcot Marie Tooth is hindfoot varus, midfoot adduction and
forefoot pronation.

Question 103
Which structure most commonly obstructs closed reduction in lateral subtalar
dislocation?
Extensor digitorum brevis tendon
Extensor digitorum longus tendon
Extensor retinaculum
Peroneus longus tendon
Tibialis posterior tendon

References
Leitner B. Obstacles to reduction in subtalar dislocations. J Bone Joint Surg Am 1954; 36: 299
http://www.wheelessonline.com/ortho/sub_talar_dislocation
Lateral dislocation is the less common type of subtalar dislocation (15%);the calcaneus is
displaced lateral to the talus. The talar head lies medially, and foot appears pronated.The
navicular
lies
lateral
to
the
talar
neck;

Lateral

dislocations

may

be

complicated

by

interposed

posterior

tibial

tendon

Lateral dislocation is the less common type of subtalar dislocation (15%);the calcaneus is
displaced lateral to the talus. The talar head lies medially, and foot appears pronated.The
navicular
lies
lateral
to
the
talar
neck;
Lateral dislocations may be complicated by interposed posterior tibial tendon

Question 104
Following isolated Posterior Cruciate Ligament injury the knee is most unstable in
which position
Extension
30 degrees flexion
60 degrees flexion
Arc from 30 to 90 degrees flexion
90 degrees flexion

References
Review of Orthopaedics, Miller
The key examination of the posterior cruciate ligament is the posterior draw test, with an
absent or posteriorly displcaed 'step-off'.

Question 105
Which of the following statements is true of Syme's amputation?
Allows for easy application of a prosthesis
Can be performed in the absence of a viable heel pad.
Can only be performed in the presence of a patent posterior tibial artery
Is preferred more in female than male patients.
Is not a good operation for children with congenital foot deformities.

References
Symes amputation is indicated for congenital foot deformities, ?bular hemimelia and severe
injury
to
the
foot
as
long
as
the
heel
pad
remains
viable.
The posterior tibial artery in the posterior ?ap must be preserved since it provides a blood supply
to
the
heel
flap.
The tough durable skin of the heel flap provides normal weight bearing skin.
The two most common causes of an unsatisfactory Syme stump are posterior migration of the
heel
pad
and
skin
sloughing.
The prosthesis used for a classic syme stump consists of a moulded plastic socket and a solid
ankle,
cushioned
heel
foot
(SACH).
The prosthesis used must accommodate the flair of the distal tibial metaphysis and thus is large
and
bulky.
Therefore
this
amputation
is
not
recommended
for
women.

Information from Campbell's Operative Orthopaedics, Page 590, Edition, Eleventh edition
Gaine WJ, McCreath SW Syme's amputation revisited a review of 46 cases. Bone Joint Surg [Br]
1996;78-B:461-7.

Question 106
Which of the following is NOT true about reverse pivot test
Can be positive in 35% of normal subjects
The ilio-tibial band should be intact to elicit a positive test
An internal rotation and valgus force is applied.
The knee is extended from flexed position
The lateral tibial condyle moves from posterior subluxation to reduction

References
1

Dan

Cooper

(JBJS)

1991

To perform a reverse pivot shift test a valgus external rotation force is applied from a flexed
position of the knee.

Question 107
A 25 year old male sustained a traumatic anterior shoulder dislocation 4 weeks ago whilst
playing rubgy. This is his third dislocation in the last 6 months. Besides an AP and axillary
lateral views what other radiological view will aid your investigation of his shoulder?
Scapular Y view
30 Degrees Caudal Tilt view
Serendipity view
Westpoint view
Hobbs view

References
Review of Orthopaedics. Sports Medicine. Section four. Miller 4th Edition page 237.
West Point view (prone with shoulder abducted) (beam directed inferior-lateral to superior-medial
at 25 degrees from patients midline, and posterior-inferior to superior-anterior at 25 degrees
from
table
surface;
film
at
superior
aspect
of
shoulder)
Similar to axillary view but shows BEST view of anterior-inferior rim of glenoid.

Question 108
Which of the following statement regarding the menisci is TRUE?
The main form of collagen is type II.

The menisci transmit 50 - 90% of the force across the knee joint in the normal knee.
The greatest concentration of mechanoreceptors are in the anterior horns.
The cell responsible for menical healing is the fibroblast.
The medial meniscus is more circular in shape than the lateral meniscus.

References
Miller. Review of Orthopaedics, 5th Edition, pages 42, 43 and 247
The main form of collagen in the meniscus is type I.

Question 109
In a patient with a suspected compartment syndrome following tibial fracture
Fasciotomies should only be performed once the diagnosis has been confirmed with
compartment pressures.
If the compartment tissue pressures are within 20mmHg of the diastolic BP fasciotomies are
not required.
In minor severity cases fasciotomies may be performed over the fracture site only.
Pain on passive toe movement is a clinical sign of compartment syndrome.
The anterior compartment always has increased pressures when a true case of
compartment syndrome is found.

References
Prompt diagnosis of an acute compartment syndrome is the key to a successful outcome.
Pain is usually the first sign of a compartment syndrome but in patients who have an altered
conscious level or are on a PCA or have had other analgesic interventions this may not be
the case.
Compartment pressures should be measured, even if this occurs in the operating theatre,
but a delay in measuring equipment availability should not delay fasciotomies being
performed in the presence of clinical signs.
Intracompartment pressures (ICP) are usually measured in the anterior compartment of the
leg as this is the compartment most commonly involved and is most easily accessible. If the
anterior compartment alone is measured and the pressures are not vastly elevated there is a
small chance of deep posterior compartment syndrome being missed.
Whitesides et al stated that there is inadequate perfusion and relative ischaemia when the
tissue pressure rises to within 10-30 mmHg of the diastolic blood pressure.
Fractures in adults, Rockwood and Green, sixth edition, chapter13
Compartment syndrome in open tibial fractures.Blick-SS; Brumback-RJ; Poka-A; BurgessAR; Ebraheim-NA J-Bone-Joint-Surg-Am. 1986 Dec; 68(9): 1348-53

www.wheelessonline.com/ortho/compartment_syndrome_resulting_from_tibial_frx
Tissue pressure measurements as a determinant for the need of fasciotomy. Whitesides TE,
Haney TC, Morimoto K et al.Clin Orthop 1975;113:43-51

Question 110
When considering the pathology related to Morton's neuroma of the foot.
A : Collagen hypertrophy
B : Degenerative neural changes
C : Electron microscopy
D : Endarteritis obliterans
E : Fusiform swelling
F : Light microscopy
G : Neuritis
H : Proliferative neural changes
I : Renaut bodies
J : Separation of nerve fibres
1 : The most typical histological feature observed
Correct answer:
B
Your answer:
B
2 : The presence of which histological feature represents a compressive pathology
Correct answer:
I
Your answer:
I
3 : The best method for observing the diagnostic pathological micro-anatomy.
Correct answer:
C
Your answer:
C

References
Klenerman L. Mini-symposium:Nerve compression syndrome (V) Nerve compression in the foot.
Current
Orthopaedics
(2001)15,275-280
The swelling seen in this condition is generally called a neuroma, but this is incorrect; histological
sections show that the neural changes are degenerative in nature, not proliferative.
The microscopic anatomy shows separation of individual nerve fibres by a proliferating
collagen matrix. The presence of Renaut bodies suggests a compressive pathology. Renaut
bodies are loosely textured, whorled, cell-sparse structures in the sub-perineural space of
peripheral nerves but this diagnostic feature is only seen on electronmicroscopy.

Question 111
What would be the best method of management of knee injuries in
A : PCL and posterolateral corner reconstruction
B : Quadriceps strengthening programme

C : Acute reconstruction of PCL


D : Open reduction and internal fixation of avulsion fracture
E : Total knee replacement
F : PCL reconstruction after ACL and MCL reconstruction
G : Urgent femoral angiogram
H : Urgent knee EUA +arthroscopy
I : Reconstruction of the anteromedial band of the PCL
1 : A 25 year old who sustained a hyperflexion injury to his knee 8 weeks ago. He has a
positive quadriceps active test and a positive dial test at 30 and 90 degrees of external
rotation. He has had quadriceps strengthening physiotherapy, but still complains of
instability
Correct answer:
A
Your answer:
A
2 : A 70 years old man sustained a dashboard injury 10 days ago. He has a positive
posterior drawer test. Radiographs do not show any obvious fractures. He did complain of
moderate knee pain for the past 3 years or so prior to the injury
Correct answer:
B
Your answer:
B
3 : A 45 year old was involved in a motorcycle accident presenting 7 days post-injury with
a swollen knee. Radiographs show an avulsion of his posterior tibial spine
Correct answer:
D
Your answer:
D

References
Review
of
Orthopaedics
-Miller,
5th
Edition,
page
258
The PCL is most commonly injured with a direct blow to the anterior tibia with the knee flexed, or
can be injured with a fall onto the ground with a plantar flexed foot. Non operative treatment can
be carried out for isolated grade I or II PCL injuries. Bony avulsion fractures can be repaired
primarily with good results.

Question 112
With respect to nerves of the lower limb
A : Common digital nerve
B : Lateral plantar nerve
C : Medial plantar nerve
D : Sural nerve
E : Deep peroneal nerve
F : Superficial peroneal nerve
G : Tibial nerve
H : Saphenous nerve
1 : This is the main innervator of extensor digitorum brevis
Correct answer:
E
Your answer:
G

2 : This is in danger at the proximal portion of the lateral approach to calcaneal


fracture fixation
Correct answer:
D
Your answer:
D
3 : This supplies the skin over the dorso-medial part of the great toe
Correct answer:
F
Your answer:
E
4 : This pierces the deep fascia 10-15 cm proximal to the lateral malleolus
Correct answer:
F
Your answer:
D

References
LAST anatomy book; Miller 5th Edition Page 363

Question 113
Which is the best proximal femoral fixation device for the following patients
A : Cemented modular hemiarthroplasty
B : Proximal femoral nail
C : Uncemented Austin Moore hemiarthroplasty
D : Bipolar cemented hemiarthroplasty
E : Cannulated screws
F : Total hip replacement
G : Intramedullary hip screw
H : Dynamic Condylar screw (DCS)
1 : A fit 59 year old with a displaced subcapital neck of femur fracture
Correct answer:
E
Your answer:
F
2 : An 85 year old, who walks with one stick, with a displaced, transcervical neck of
femur fracture.
Correct answer:
A
Your answer:
D
3 : An active 72 year old with a displaced subcapital neck of femur fracture with
associated osteoarthritis and mini mental score of 10/10.
Correct answer:
F
Your answer:
F

References

THR has better functional results as compared to Bipolar hemiarthroplasty in healthy, active
and lucid patients (1). Internal fixation is the method of choice in young patients with
displaced intra-capsular fractures (2). Reports have shown that bipolar hemiarthroplasty
functions as a unipolar device within three to 12 months after surgery (3). Cemented
hemiarthroplasty has less post-operative thigh pain than uncemented (4).
1)Blomfeldt R, Trnkvist H, Eriksson K, Sderqvist A, Ponzer S, Tidermark J. A randomised
controlled trial comparing bipolar hemiarthroplasty with total hip replacement for displaced
intracapsular fractures of the femoral neck in elderly
patients.J Bone Joint Surg Br. 2007 Feb;89(2):160-5.
2)Bhandari M, Devereaux PJ, Tornetta P 3rd, et al. Operative management of displaced
femoral neck fractures in elderly patients: an international survey. J Bone Joint Surg [Am]
2005;87-A:212230
J Bone Joint Surg Br. 2007 Feb;89(2):160-5
(3)Phillips TW. The Bateman bipolar femoral head replacement: a fluoroscopic study of
movement over a four-year period. J Bone Joint Surg [Br] 1987;69-B:7614
(4)Parker MJ, Gurusamy K.Arthroplasties (with and without bone cement) for proximal
femoral fractures in adults.
Cochrane Database Syst Rev. 2006 Jul 19;3:CD001706. Review

Question 114
Of the 10 listed options which anatomical structure is most at risk during the
following surgical approaches for irrigation and debridement of the ankle
A : Peroneus tertius tendon
B : Extensor tendons
C : Sural nerve
D : Great saphenous vein
E : Medial plantar nerve
F : Dorsalis pedis artery
G : Superficial peroneal nerve
H : Tibialis anterior tendon
I : The knot of Henry
J : Flexor hallucis longus tendon
1 : 1. Posterolateral approach
Correct answer:
C
Your answer:
C
2 : 2. Anterior approach
Correct answer:
G
Your answer:
H
3 : 3. Posteromedial approach
Correct answer:
J
Your answer:
D

References

Campbell's Operative Orthopaedics

Question 115
Regarding the Ponseti method of correction of congenital talipes equinovarus deformity
A : Depression of the first ray
B : Correction of equinus
C : Correction of varus
D : Correction of valgus
E : Lengthening of the achilles tendon
F : Elevation of the 1st ray
G : Tibialis posterior tendon transfer
H : Tibialis anterior tendon transfer
I : Achilles tenotomy
J : Plantar fascia release
1 : The first step in deformity correction with this technique is
Correct answer:
F
Your answer:
F
2 : Surgical procedure required in approximately 90% of cases
Correct answer:
I
Your answer:
I
3 : Surgical procedure required in up to 20% of cases for correction of residual deformity
Correct answer:
H
Your answer:
H

References
Clinical Orthopaedics and Related Research, volume 467, May 2009

Question 116
Regarding stability of the knee
A : Anterior cruciate ligament
B : Iliotibial band
C : Popliteal oblique ligament
D : Superficial medial collateral ligament
E : Deep medial collateral ligament
F : Lateral collateral ligament
G : Medial meniscus
H : Lateral meniscus
I : Posterior cruciate ligament
J : Posterolateral corner
1 : Is the primary restraint to valgus stress at all angles
Correct answer:
D
Your answer:

D
2 : Is the primary restraint to internal rotation
Correct answer:
A
Your answer:
A
3 : Is the predominant secondary restraint to anterior translation of the tibia on the femur
Correct answer:
B
Your answer:
B

References
Stable knee movement is achieved through primary and secondary restraints to knee movement.
The primary restraint to anterior translation of the tibia on the femur and internal rotation is the
ACL. Secondary restraints to anterior translation have been quantified as being the iliotibial band
(24%), mid-medial capsule (22%), mid-lateral capsule (20%), MCL (16%), LCL (12%) and the
menisci. The superficial MCL is the primary restraint to valgus stress at all angles. The deep MCL
has little resistance to valgus force. The ACL acts as a secondary restraint to valgus load.
Freeman, MA, Pinskerova, V. The movement of the normal tibio-femoral joint. J Biomech
2005;38:197-208.

Question 117
With regard to benign bone and soft tissue tumours, what is the most likely diagnosis in
the following patients?
A : Chondroblastoma
B : Enchondroma
C : Giant cell tumour
D : Glomus tumour
E : Histiocytosis X
F : Mucous cyst
G : Neurofibroma
H : Osteoblastoma
I : Osteochondroma
J : Osteoid Osteoma
1 : A 62 year old lady presents with a slowly growing swelling overlying the dorsum of her
index finger DIP joint. Radiographs show features of osteoarthritis in the hand and
erosion of the distal phalanx.
Correct answer:
F
Your answer:
F
2 : A 25 year old woman who has recently had a baby presents with a 6 month history of
progressively increasing pain and now, a swelling in her wrist. Radiographs show a pure
lytic lesion at the distal end of the radius involving the metaphysis and epiphysis but not
the articular surface.
Correct answer:
C
Your answer:
C

3 : An 8 year old boy presents with a painful swelling over his ring finger. The pain wakes
him up at night and radiographs show a well circumscribed, rounded, sclerotic lesion, < 1
cm in size with a central radiolucent nidus.
Correct answer:
J
Your answer:
J

References
Although some of the options are bony tumours, all the provided options may present with
swelling. The scenarios in the question describe patients with extremity swelling as a presenting
feature in some of the more important and commonly encountered benign lesions.
Explanation

of

answers

to

the

scenarios:

1. G Erosion of the distal phalanx is due to pressure effect. Typically, osteophytes at the DIP joint
are found underlying this cystic lesion. Enchondroma, glomus tumor, osteoid osteoma and giant
cell tumor of tendon sheath can also present in the fingers, but age and features of osteoarthritis
are
characteristic.
2. F Giant cell tumour of bone typically affects the epiphysis of long bones, the distal radius being
the 4th common site. Purely lytic lesion is characteristic and usually, this is limited to the
subchondral margin but can fracture into the joint later. Growth increases during pregnancy.
3. A Another finger swelling, but age, night pain (relieved by NSAIDS) and radiological features
are distinguishing. Size < 1 cm differentiates an osteoid osteoma from an osteoblastoma.
1.
Oxford
Textbook
of
Orthopaedics
and
2. Review of Orthopaedics, Miller 4th Ed. P 444-494

Trauma

Volume

Question 118
Which component of the physis is affected in each of the following conditions
A : Perichondrial ring of LaCroix
B : Hypertrophic layer
C : Reserve zone
D : Zone of provisional calcification
E : Proliferative layer
F : Groove of Ranvier
1 : Rickets
Correct answer:
D
Your answer:
A
2 : Gaucher's disease
Correct answer:
C
Your answer:
D
3 : Achondroplasia
Correct answer:
E
Your answer:

151-166

References
Stanmore

guide

to

basic

sciences,

paediatric

orthopaedic

secrets

Osteomalacia results from inadequate mineralisation of bone matrix (osteoid). In rickets there is
inadequate mineralisation of cartilage matrix (chondroid) and this affects the provisional zone of
calcification in the physis. Rickets is caused by a lack of serum calcium and phosphate,
insufficient to allow mineralisation of the newly formed chondroid matrix. This may be seen on
plain radiographs as a widened, thickened physis with metaphyseal flaring due to the persistence
of metaphyseal cartilage.

Question 119
The 3 scenarios below are of patients attending your revision ACL clinic. Out of the 10
options below which answer best explains the cause of ACL failure?
A : Tibial tunnel placed anteriorly
B : Tibial tunnel placed posteriorly
C : Tibial tunnel placed medially
D : Tibial tunnel placed laterally
E : Tibial tunnel placed inferiory
F : Femoral tunnel placed anteriorly
G : Femoral tunnel placed posteriorly
H : Femoral tunnel placed vertically
I : Femoral tunnel placed medially
J : Femoral tunnel placed laterally
1 : A 30 year old basketball player underwent autologous ipsilateral single bundle
hamstring ACL reconstruction. 6 months later examination reveals that he is tight in
flexion (ROM 0-100 deg) and has laxity in extension.
Correct answer:
F
Your answer:
F
2 : A 25 year old female skier underwent a BTB ACL reconstruction 4 months ago.
Examination reveals fixed flexion of 10 deg with flexion up to 110 deg. What is the cause
for her reduced ROM of movement.
Correct answer:
A
Your answer:
A
3 : A 22 year old footballer has had a BTB ACL recostruction 3 months ago and
examination reveals that he is tight in extension and lax in flexion.
Correct answer:
G
Your answer:
G

References
Christopher D. Harner, J. Robert Giffin, Roger C. Dunteman, Christopher C. Annunziata, and
Marc
J.
Friedman
Evaluation and Treatment of Recurrent Instability After Anterior Cruciate Ligament
Reconstruction*

J.
Bone
Joint
Surg.
Am.,
Nov
2000;
82:
1652.
The paper by Harner et al states that inappropariate tunnal positioning results in excessive
changes in graft length as the knee moves through its range of movement. Only small changes
in length can be accomodated before undegoing plastic deformation, therefore a malposiitoned
graft can either lead to loss of motion or lengthening of the graft over time. Anterior femoral
tunnel placement is the most common error during ACL reconstruction.

Question 120
Concerning the muscle layers of the sole of the foot
A : Layer 1
B : Layer 2
C : Layer 3
D : Layer 4
E : Layer 1+3
F : Layer 2+4
G : Layer 4+5
H : Layer 3+5
I : Between first and the second layer underneath flexor digitorum brevis.
J : Between second and third layer protected by quadratus plantae
1 : In what layer do medial and lateral plantar nerves lie
Correct answer:
I
Your answer:
G
2 : Flexor digitorum longus and lumbricals are part of
Correct answer:
B
Your answer:
F
3 : Peroneus longus and interossei are part of
Correct answer:
D
Your answer:
E

References
1. Online grays anatomy.
Knowing layers of foot is important for those involved in foot surgery.
First layer consists (from lateral to medial) of the ADM, FDB, & abductor hallucis.
Medial and lateral Plantar nerve and artery lies between First and second layer.
Second layer (from medial to lateral) tendon of FHL, tendon of FDL, quadratus plantae &
lumbricalsin
Third layer has adductor hallucis, & FHB
Fourth layer has Palmer and Dorsal interossei.
First two layers originate from calcaneal tuberosity and the other two from the metatarsal
shafts.
Lateral release of first MTP joint in an integral part of Bunion surgery. It involves release of
Adductor hallucis which lies in the third layer and often surgeon ends up just releasing the
dorsal interossei which from the dorsal side is superficial to adductor hallucis and is in layer
4. Flexor digitorum longus is used for tendon transfer in acquired flat foot correction.

Question 121
Choose the most appropriate treatment for the following scenarios in Developmental
Dysplasia of the Hip:
A : Pavlik harness
B : Double Nappies
C : Medial open reduction and hip spica
D : Salter innominate osteotomy
E : Dega Osteotomy
F : Chiari Osteotomy
G : Ganz osteotomy
H : Total hip replacement
I : Broomstick plaster
1 : 6 week old girl with a reducible hip dislocation
Correct answer:
A
Your answer:
A
2 : 5 year old with a concentric hip and an acetabular index of 35 degrees
Correct answer:
D
Your answer:
D
3 : 14 year old girl with a lateralized, spherical but non-concentric hip
Correct answer:
F
Your answer:
F

References
Tachdjian's Pediatric Orthopaedics. 4th Edition. 2008. Developmental Dysplasia of the Hip: pp
637-770.
Pavlik Harness is the treatment of choice for child 1-6 months of age with DDH. The harness
must hold the hip in more than 90 degrees of flexion with the proximal femoral metaphysis
pointed towards the triradiate cartilage. If reduction is not obtained or maintained (clinically and
ultrasonographically) within 3-4 weeks, the harness is discontinued. If reduction is confirmed, the
harness
is
continued
for
6
weeks
after
stability
is
established.
Salter or Pemberton osteotomy is recommended for patients younger than 8 years of age with
concentric but dysplasic hips. After 2 years of age an acetabular index over 30 degrees is
definitely abnormal. There is good evidence that acetabular dysplasia persisting beyond 5 years
of
age
does
not
adequately
correct
and
requires
a
pelvic
osteotomy.
A Chiari osteotomy or a shelf procedure (eg. Staheli) is warranted in non-concentric hips. These
are salvage procedures since the femoral head is eventually covered by fibrocartilage and not
repositioned acetabular cartilage.

Question 122
With regards to the nerve supply to the foot choose the nerve that innervates each of the
structures below.
A : Common peroneal

B : Deep Peroneal Nerve


C : Lateral Plantar nerve
D : Medial plantar nerve
E : Posterior tibial
F : Superficial peroneal nerve
G : Sural nerve
H : Tibial nerve
1 : Flexor digitorum brevis
Correct answer:
D
Your answer:
F
2 : Abductor Hallucis
Correct answer:
D
Your answer:
B
3 : First web space sensation
Correct answer:
B
Your answer:
B

References
Sinnatamby.C,
Lasts
anatomy
regional
and
applied
Miller.M,
Review
of
Orthopaedics
The nerve supply to the foot is essential knowledge. The cutaneous supply of the nerves running
through each compartment of the leg is also essential knowledge for the assessment of the
patient
presenting
with
a
compartment
syndrome.
Flexor digitorum longus (FDL) is in the second plantar layer of the foot and enters the sole on the
medial side of the tendon of flexor hallucis longus (FHL), it divides into four tendons as it crosses
superficial to the FHL tendon.The four tendons pass forward on the sole deep to those of flexor
digitorum brevis (FDB) and after giving off the lumbricals they enter the fibrous sheaths of the
lateral four toes. Each tendon perforates the tendon of FDB and inserts into the base of the distal
phalanx. Its function is to flex the lateral four toes in any position of the ankle joint. FDB is
supplied
by
the
medial
plantar
nerve.
Abductor hallucis arises from the medial process of the calcaneus and from the flexor
retinaculum. It runs along the medial border of the foot before inserting into the medial side of the
base of the proximal phalanx of the great toe, Its function is to abduct the great toe. Its nerve
supply
is
the
medial
plantar
nerve.
The deep peroneal nerve runs through the anterior compartment of the leg and supplies
sensation to the first web space.

Question 123
Which of the following tendons is the most appropriate donor tendon for transfer in the
following scenarios:
A : Tendo Achilles
B : Peroneus brevis
C : Flexor hallucis longus

D : Flexor digitorum longus


E : Tibialis posterior
F : Tibialis anterior
G : Extensor digitorum longus
H : Extensor hallucis longus
I : Peroneus longus
1 : A 54 year old man who felt a sudden pain in his calf while playing football 15 months
ago and has subsequently had ongoing weakness of ankle plantarflexion.
Correct answer:
C
Your answer:
C
2 : A 48 year old woman with a four year history of a progressive, painful, mobile flatfoot
deformity
Correct answer:
D
Your answer:
D
3 : A 63 year old man who had an episode of severe back pain a year ago and who now
has residual alteration in sensation on the medial border of the calf and a high stepping
gait with flapping foot.
Correct answer:
E
Your answer:
E
4 : A 32 year old woman with flexible clawing of the hallux and a painful, tender plantar
callosity overlying a prominent first metatarsal head
Correct answer:
H
Your answer:
G

References
Coughlin,
Mann
and
Saltzmann.
Surgery
of
the
Foot
and
Ankle.
The first scenario describes a chronic Achilles tendon rupture. While a tendon transfer is not
mandatory, most surgeons would use FHL as first preference as it is easily accessed and works
in
phase.
The second scenario describes a grade 2 tibialis posterior tendon dysfunction. FDL is the tendon
transfer of choice as the tendon is found alongside tibialis posterior and functions in phase.
This patient has had a far lateral L4/L5 disc prolapse affecting the L4 nerve root, leaving a
footdrop. Tibialis posterior is innervated by L5 and so is still functional for a transfer.
This is the indication for the Jones procedure, namely hallux interphalangeal arthrodesis and
transfer of EHL into the distal metatarsal.

Question 124
Approximate age at which each of the following ossification centers appear?
A : 2 months
B : Birth
C : 10 years

D : 6 months
E : 3 years
F : 1 year
G : 5th week gestation
H : 16 years
1 : Olecranon
Correct answer:
C
Your answer:
C
2 : Clavicle
Correct answer:
G
Your answer:
G
3 : Greater tuberosity of the humerus
Correct answer:
E
Your answer:
E

References
MIller table 2.1 (p134) Ossification Centres

Question 125
Tumours around the knee in the young adult. Select the most appropriate first line
treatment modality of each of the following conditions
A : Above knee amputation
B : Intra-articular chemotherapy
C : Marginal resection
D : Neoadjuvent chemotherapy
E : Radical resection
F : Radiotherapy
G : Wide local excision with endoprosthetic replacement
H : Wide local excision with joint preservation
I : Intra-lesional procedure
1 : Enneking IIa Osteosarcoma of distal femoral metaphysis
Correct answer:
D
Your answer:
D
2 : Localised Nodular PVNS
Correct answer:
C
Your answer:
C
3 : Large Distal femoral Aneurysmal Bone Cyst
Correct answer:
I
Your answer:
I

References
Osteosarcomas are primarily treated with Neoadjuvent chemotherapy as this is the most
important treatment modality for improving survival. This is thought to reduce the presence
of early pulmonary micrometastasis.
Nodular PVNS can be resected locally unless it has breached the joint capsule. There is
however a high recurrence rate and re-resections are often combined with adjuvent
radiotherapy.
Aneurysmal bone cysts and Giant Cell Tumours can be treated with curettage and impaction
bone grafting - a form of intralesional resection.

Question 126
Which of the following muscles must routinely be detached (in part, or in full) to facilitate
the approach stated?
A : Vastus Medialis
B : Rectus Femoris
C : Obturator Externus
D : Piriformis
E : Quadratus Femoris
F : Vastus Lateralis
G : Gluteus Medius
H : Sartorius
I : Psoas
1 : Anterior approach (Smith-Peterson) to the hip?
Correct answer:
B
Your answer:
B
2 : Posterior approach to the hip?
Correct answer:
D
Your answer:
D
3 : Direct lateral approach (Hardinge) to the hip?
Correct answer:
G
Your answer:
G

References
Surgical Exposures in Orthopaedics. Hoppenfeld, de Boer, Third Edition. Lippincott Williams &
Wilkins
Smith Peterson Approach - This approach utilises the internervous plane superficially between
sartorius and tensor fascia lata and more deeply rectus femoris and gluteus medius. The rectus
is
detached
in
part
during
part
of
the
exposure.
Posterior Approach - This approach involves detachment of the piriformis muscle and release of
the
short
external
rotators
to
gain
access
to
the
hip
joint.

Watson - Jones - This approach utilises the interval between tensor fascia lata & gluteus medius.
Gluteus medius can be detached in part during part of the exposure

Question 127
With regards to the cruciate and collateral ligaments of the knee, which structure need to
be damaged to result in
A : Superficial MCL
B : Anterior Cruciate Ligament
C : Posterior Cruciate Ligament
D : Deep Part of MCL
E : Lateral Collateral ligament
F : Popliteus Tendon
G : Medial Meniscus
H : Lateral meniscus
I : Lateral Meniscofemoral Ligament
J : Gastronemius Tendon
1 : Loss of roll-glide mechanism
Correct answer:
B
Your answer:
C
2 : Positive Valgus stress test
Correct answer:
A
Your answer:
D
3 : Loss of screw home mechanism and increased tibial external Rotation
Correct answer:
F
Your answer:
F

References
Goldblatt JP, Richmond JC. Anatomy and biomechanics of the knee. Operative techniques in
Sports
Medicine,2003;
11(3):
172-186.
Martelli S, Pinskerova V. The shapes of the tibial and femoral articular surfaces in relation to
tibiofemoral
movement.
Freeman MAR, Pinskerova V. The movement of the normal tibio-femoral joint. J Biomech,2005;
38:197-208.

Question 128
The following pathologies are associated with which of these clinical conditions?
A : Cerebral Palsy
B : Rett's syndrome
C : Poliomyelitis
D : Guillain-Barre syndrome
E : Charcot-Marie-Tooth Disease
F : Freidrich's Ataxia
G : Spinal Muscular Atrophy

H : Duchenne's Dystrophy
I : Becker's Dystrophy
J : Werdnig-Hoffmann Disease
1 : Genetically determined demyelination and/or axonal degeneration in peripheral
nerves
Correct answer:
E
Your answer:
H
2 : Periventricular Leukomalacia
Correct answer:
A
Your answer:
I
3 : Autoimmune mediated demyelination and/or axonal destruction in peripheral
nerves
Correct answer:
D
Your answer:
D

References
Tachdjian's Pediatric Orthopaedics. 4th Edition, 2008. Vol 2, Neuromuscular Disorders: pp
1675-1674.
CMT has been classically divided into demyelinating and axonal forms. But research
indicates that demyelination renders the axon susceptible to degeneration and henec the 2
pictures can co-exist.
Periventricular leukomalacia and intra and periventricular haemorrhages are frequent MRI
finidings in Cerebral palsy. The former results from an ischemic insult to the arterial
watershed area close to the ventricular walls.
GB Syndrome is now the commonest cause of acute flaccid paralysis in children in the west.
It is characterised by symmetric motor and sensory paresis of the limbs and at times the
trunk. The disease is auto-immune and directed against peripheral nervous system myelin,
axon or both. It is triggered by a preceding bacterial or viral infection.

Question 129
Concerning lumbar intervertebral discs
A : Type III Collagen
B : Type VI Collagen
C : Type II Collagen
D : Type IV Collagen
E : Type X Collagen
F : Type I Collagen
G : Type XI Collagen
H : Type IX Collagen

1 : Which type of collagen is most prevalent in the nucleus pulposus affected by age
related change?
Correct answer:
F
Your answer:
F
2 : Which type of collagen is most prevalent in the nucleus pulposus of the normal
disc?
Correct answer:
C
Your answer:
F
3 : Which type of collagen is involved in the cross linking of aggregates in the
intervertebral disc?
Correct answer:
H
Your answer:
B

References
1. Hadjipavlou AG, Tzermiadianos MN, Bogduk N, Zindrick MR. The pathophysiology of disc
degeneration: a critical review. J Bone Joint Surg Br 2008;90-10:1261-70.
1. The collagen content of the nucleus increases and changes from type II to type I Collagen
rendering the nucleus more fibrous during the ageing process[1]. The concentration of cells
and proteoglycans however decrease within the disc with age.
2. Type IX collagen cross links aggregates which are held together by type II collagen.

Question 130
When designing a study which factor needs to be considered if
A : Power analysis
B : Null hypothesis
C : Parametric test
D : Type II error
E : Type I error
F : Linear regression
G : Independent variables
H : Matching
I : Randomisation
1 : You are trying to calculate the sample size needed
Correct answer:
A
Your answer:
A
2 : You are choosing the rate of false positive errors
Correct answer:
E
Your answer:
D
3 : You are comparing two continous variables

Correct answer:
F
Your answer:
F

References
A power analysis is a method of determining the number of patients required in a study to have a
reasonable
chance
of
showing
a
difference
if
one
exists.
The null hypothesis is an assumption that any difference seen is purely by chance. Studies are
designed
to
either
prove
or
disprove
this
assumption.
A parametric test assumes data is sampled from a particular form of distribution such as a normal
distribution.
Non-parametric
tests
make
no
such
assumption
Errors
arise
when
accepting
or
rejecting
the
null
hypothesis.
A type I (alpha) error occurs when a difference is found but in reality there is not a difference. A
type II (beta) error occurs when no difference is found but a difference does exist.
Linear regression Correlation is a term used to describe the relationship between two
parameters. Linear regression is when the relationship can be plotted on a straight line such as
with
parametric
data.
Regression
can
also
be
curved
or
logistic.
For variables to be independent, there needs to be no chance that a subject could appear in both
groups being compared. An unpaired T test would be used to compare independent variables
provided
they
follow
a
normal
distribution.
Matching is a process of identifying subjects in different groups that have certain similar
characteristics
(eg.
Age,
sex,
co-morbidities)
Randomisation ensures that all prognostic variables, known and unknown, will be distributed
evenly among the treatment groups. Randomisation can be simple (eg. Computer-generated
tables),
stratified,
or
block.
Basic Orthopaedic Sciences. The Stanmore Guide. Ramachandran M. Hodder Arnold.

Question 131
Regarding the treatment of L5/S1 spondylolithesis:
A : L5-S1 posterolateral fusion
B : Regular review and repeat radiographs
C : In situ posterolateral fusion including L4 in the arthrodesis
D : Anterior fusion alone
E : Reduction of the spondylolisthesis
F : Physiotherapy
G : Facet joint injection
H : Kyphoplasty
1 : A grade II slip in an asymptomatic adolescent patient should be managed by
Correct answer:
B
Your answer:
B
2 : A grade II slip in a patient with persistent back pain should be managed by
Correct answer:

A
Your answer:
A
3 : A grade II slip in a patient with progressive slips should managed with
Correct answer:
A
Your answer:
A
4 : A grade III slip is best treated by
Correct answer:
C
Your answer:
C

References
Dubousset J: Treatment of spondylolysis and spondylolisthesis in children and adolescents. Clin
Orthop
337:77-85
Hensinger RN: Current concepts review: Spondylolysis and spondylolisthesis in children and
adolescents.
JBJS
1989
71A:
1098-1107.
L5/S1 fusion is the gold standard for patients with grade II slips with progressive slips, persistent
back
pain,
neurological
deficits.
With grade III slip, the treatment of choice is an insitu posterolateral fusion including L4 in the
arthrodesis. Anterior fusion alone, in the absence of posterior column stabilisation, is not
biomechanically stable enough.

Question 132
With regard to nerves at risk during surgical approaches to the hip:
A : Pudendal nerve
B : Nerve to obturator internus
C : Superior gluteal nerve
D : Inferior gluteal nerve
E : Sciatic nerve
F : Posterior femoral cutaneous nerve
G : Lateral femoral cutaneous nerve
H : Nerve to quadratus femoris
I : The cluneal nerves
J : The anterior and posterior divisions of the obturator nerve
1 : Which nerve may be injured during a medial approach?
Correct answer:
J
Your answer:
B
2 : Which nerve may be injured during a anterior approach?
Correct answer:
G
Your answer:
B
3 : Which nerve may be injured during a lateral approach?
Correct answer:

C
Your answer:
C

References
Hoppenfeld S, deBoer P. Surgical exposures in orthopaedics the anatomic approach. 3rd ed.
Lippincott
Williams
and
Wilkins
2003
The anatomy of the medial approach is the anatomy of the adductor compartment of the thigh.
The obturator nerve is derived from the anterior division of the L2-L4 nerve, and divides in the
obturator notch into anterior and posterior divisions. The nerve lies on the anterior surface of the
adductor brevis. The posterior division of the obturator nerve runs dially on the surface of the
adductor
magnus.
The anterior approach internervous plane lies between the sartorius and the tensor fascia lata.
The lateral femoral cutaneous nerve passes either over, behind or through the sartorius muscle.
The lateral approach allows exposure of the hip joint for joint replacement. The superior gluteal
nerve runs between the gluteas medius and minimus 3-5cm above the greater trochanter and
can be damaged with proximal dissection.

Question 133
With regards to bone tumours choose the most appropriate response for each of the
following
A : Osteosarcoma
B : Chondrosarcoma
C : Enchondroma
D : Ewings sarcoma
E : Osteoid osteoma
F : Unicameral bone cyst
G : Non-ossifying fibroma
H : Giant cell tumour
I : Chondroblastoma
J : Metastatic bone tumour
1 : Associated with Paget disease
Correct answer:
A
Your answer:
A
2 : Lytic lesion in adults that can extend to subchondral area, narrow zone of transition
Correct answer:
H
Your answer:
H
3 : Mirel's score
Correct answer:
J
Your answer:
J
4 : Can be treated by radio-frequency ablation
Correct answer:
E

Your answer:
B

References
Apley's
Miller
Ramachandran Basic Sciencs

Question 134
For each of the following patients select the most appropriate diagnosis from the list?
A : Labral tear
B : Meralgica paraesthetica
C : Rectus femoris avulsion tear
D : Psoas tendonitis
E : Psoas haematoma
F : Hamstring avulsion
G : Gluteus medius avulsion tear
H : Ruptured ligamentum teres
I : Cam impingement
J : Trochanteric bursitis
1 : A 65 year old patient presents with right groin and thigh pain without an injury. He has
a mechanical aortic valve. His knee gives way and he has noticed numbness over the
lower anterior thigh and upper shin.
Correct answer:
E
Your answer:
E
2 : A patient has been treated for lateral right hip pain. This was particularly
troublesome going up and down stairs and lying on the left side. She was told she had a
bursitis. Multiple cortisone injections to the outer side of the hip were performed in the
clinic over a period and although initially helpful the symptoms have now changed to pain
with weight bearing and a limp.
Correct answer:
G
Your answer:
A
3 : A patient following a cemented Charnley right total hip replacement has never been
100% improved. The severe nocturnal pain has been relieved but the patient has new
annoying groin pain. The patient has noticed when driving a car she has to lift the right
leg out of the car to limit groin discomfort.
Correct answer:
D
Your answer:
G

References
Leong K, Wright, JE. An unusual case of difficulty walking: A psoas haematoma. Geriatric
Medicine
2008;38:664-665.
Pain
related
to
the
psoas
muscle
after
total
hip
replacement
V. Jasani, P. Richards, C. Wynn-Jones. J Bone Joint Surg [Br] 2002;84-B:991-3.

Operative repair of bilateral spontaneous gluteus medius and minimus tendon ruptures. A case
report. Fisher DA, Almand JD, Watts MR. J Bone Joint Surg Am. 2007 May;89(5):1103-7.
Gluteus medius tendon tears and avulsive injuries in elderly women: imaging findings in six
patients. Chung CB, Robertson JE, Cho GJ, Vaughan LM, Copp SN, Resnick D.
AJR
Am
J
Roentgenol.
1999
Aug;173(2):351-3.
Spontaneous iliopsoas hematoma is a rare but well-recognized complication of anticoagulation
that may result in significant neurological disability due to pressure on the Femoral nerve.
Gluteus medius tendon rupture can present as debilitating lateral hip pain. It is also commonly
known as Rotator cuff tear of the hip. It can be misdiagnosed as trochateric bursitis. Sometimes
steroid
injection
can
be
a
predisposing
factor.
Anterior groin pain following THR could be related to Psoas tendonitis. Typically aggravated by
certain activities like standing form low chairs.

Question 135
A ten year old boy presents with symptomatic rigid pes planus requiring excision of a
tarsal coalition
A : Sural nerve
B : Extensor digitorum longus
C : Superficial peroneal nerve
D : Talus and calcaneum
E : Extensor digitorum brevis
F : Sustentaculum tali
G : Calcaneum and cuboid
H : Calcaneum and navicular
I : Medial malleolus and talus
J : Deep peroneal nerve
1 : Through Ollier's approach, which structure(s) is/are most at risk
Correct answer:
C
Your answer:
J
2 : An osseous bar between which structures is most likely
Correct answer:
H
Your answer:
H
3 : Interposition of which structure(s) completes the procedure
Correct answer:
E
Your answer:
E

References
Campbells operative orthopaedics vol 2 chapter 38
Disorders of the Foot and Ankle Jahss Vol 1 chapter38

Question 136

For each of the following clinical scenarios, select the MOST likely anatomical
abnormality
A : Anterior glenoid rim fracture
B : Bankart lesion
C : Brachial plexus injury
D : Greater tuberosity fracture
E : Hill-Sachs defect
F : Long head of biceps rupture
G : Reverse Hill-Sachs defect
H : Rotator cuff tear
I : SLAP lesion
J : Axillary nerve palsy
1 : A 70-year old man with ongoing shoulder pain and weakness, 4 weeks following 1st
time traumatic anterior shoulder dislocation
Correct answer:
H
Your answer:
H
2 : A 23-year old male rugby player with recurrent anterior shoulder dislocation
Correct answer:
B
Your answer:
B
3 : A 45-year old male with a dislocated shoulder following an electric shock
Correct answer:
G
Your answer:
J

References
Rotator cuff tears must be considered in association with anterior dislocation with advancing age
as their incidence increases with age. The Bankart lesion is the most common pathology
implicated in recurrent anterior shoulder instability.The patient with seizure had aposterior
dislocation and therefore a reverse Hills sach lesion is the most likely abnormality
F.A. Matsen, R.M. Titelman and S.B. Lippitt et al., Glenohumeral instability. In: C.A. Rockwood,
F.A. Matsen and M.A. Wirth, Editors, The shoulder (3rd ed.), Saunders, Philadelphia (2004), p.
686
Matsen et al Practical evaluation and management of The Shoulder, 1994, pg102
Campbells

Operative

Orthopaedics

11th

edition,

pg

2679

MD Miller review of orthopaedics, 4th ed, pg 531

Question 137
What is the most appropriate step to do next in the following patients with resurfacing
hip arthroplasty?
A : Reassure and review in a year
B : Revise both components
C : Convert to a femoral stem and retain the cup

D : Perform diagnostic aspiration


E : Discharge and advise patient to return only if there is new symptom
F : Order DEXA scan to look for femoral neck osteopenia
G : Organise a CT scan of the hip
H : Order a bone scan
I : Check cobalt and chromium levels and arrange cross sectional imaging.
J : Perform chromium and cobalt levels in three months.
1 : 4 years postoperatively, patient attends routine review with no complaint.
Correct answer:
A
Your answer:
E
2 : A 60 yr old gentleman 2 yr following resurfacing has increasing pain in the groin.
Correct answer:
I
Your answer:
H
3 : A 40yr old man who has a malpositioned resurfacing prosthesis has cobalt levels showing 9 ppb chromium levels.
Correct answer:
J
Your answer:
J

References
MHRA MEDICAL DEVICE ALERT ISSUED 22 APRIL 2010
For patients implanted with MoM hip replacements:
follow up patients at least annually for five years postoperatively and more frequently in the
presence of symptoms. Beyond five years, follow up in accordance with locally agreed
protocols
investigate patients with painful MoM hip replacements. Specific tests should include
evaluation of cobalt and chromium ion levels in the patients blood and cross sectional
imaging including MRI or ultrasound scan
consider measuring cobalt and chromium ion levels in the blood and/or cross sectional
imaging for the following patient groups:
> patients with radiological features associated with adverse outcomes including
component position
> patients with small component size (hip resurfacing arthroplasty only)
> cases where the patient or surgeon is concerned about the MoM hip replacement
> cohorts of patients where there is concern about higher than expected rates of failure
if either cobalt or chromium ion levels are elevated above seven parts per billion (ppb),
then a 2nd test should be performed three months after the first in order to identify patients
who
require closer surveillance, which may include cross sectional imaging

if imaging reveals soft tissue reactions, fluid collections or tissue masses then consider
revision surgery

Question 138
Which anatomical structure around the shoulder is being discussed in each
question?
A : Transverse Humeral Ligament
B : Acromioclavicular Ligaments
C : Coracoacromial Ligament
D : Coracoclavicular Ligaments
E : Coracohumeral Ligament
F : Rotator Interval
G : Posterior Capsule
H : Fasciculus obliquus
I : Long head of biceps
1 : Primarily resists anteroposterior translation of the acromioclavicular joint
Correct answer:
B
Your answer:
B
2 : Is bound medially by the lateral coracoid base, superiorly by the anterior edge of
supraspinatus and inferiorly by the superior border of subscapularis
Correct answer:
F
Your answer:
F
3 : Prevents inferior translation of the coracoid and acromion from the clavicle
Correct answer:
D
Your answer:
D

References
Miller MD, 'Review of Orthopaedics' (4th Ed), pg 233

Question 139
With respect to replantation select the best answer
A : Transport digit cooled to 5 degree C
B : Bilateral midlateral skin incisions
C : Isolate nerves & vessels
D : Achieve skeletal fixation
E : Arterial anastomosis
F : Venous anastomosis - 2 veins per artery
G : Venous anastomosis - 2 veins only
H : Administer systemic heparin
I : Assessment using pulse oximeter monitoring
J : Loosen dressings
K : Return to operating theatre
1 : When performing a digit replantation which vascular anastomsis should be done first?

Correct answer:
E
Your answer:
F
2 : Which structures should ideally be repaired/fixed prior to vascular repair?
Correct answer:
D
Your answer:
D
3 : You are called to review a post-op replant's viability, what is the most appropriate first
step?
Correct answer:
J
Your answer:
I

References
Greens operative hand surgery. chapter 45

Question 140
With regards neuromuscular disorders which diagnosis fits best with the following
patients?
A : Dermatomyositis
B : Motor neurone disease
C : Polymyositis
D : Multiple sclerosis
E : Hunters syndrome
F : Duchennes muscular dystrophy
G : Friedrichs ataxia
H : Systemic lupus erythematous
I : Amyotrophic lateral sclerosis
J : Psoriasis
1 : A 5 year old boy with progressive weakness with difficulty in standing up.
Correct answer:
F
Your answer:
D
2 : A 15 year old girl with a progressive staggering or stumbling gait with frequent falls
Correct answer:
G
Your answer:
G
3 : A 50 year old lady presenting with pain, weakness and skin rash
Correct answer:
A
Your answer:
H

References

Apley's
System
of
Orthopaedics
and
fractures,
8th
edition
Duchennes muscular dystrophy: x-linked inheritance. condition unsuspected until child walks.
difficulty in standing and climbing stairs. characterstic feature - method of rising from floor by
climbing
up
the
legs
(Gowers'
sign).
Friedrichs ataxia: spinocerebellar dysfuntion. progressive muscle weakness of lower limb and
trunk.
degeneration
of
posterior
root
ganglia
and
peripheral
nerves.
Dermatomyositis: proximal, symmetric muscle weakness, elevation of serum muscle enzymes
characteristic electromyographic abnormalities and compatible skin involvement.

Question 141
With regards to clinical tests for diagnosing hand pathologies please select the
correct response for each
A : Inferior Radioulnar subluxation
B : Carpal Translocation
C : Metacarpophalangeal joint subluxation and ulnar drift
D : Boutonniere deformity
E : Swan neck deformity
F : Carpometacarpal arthritis
G : Z deformity of thumb
H : Mallet finger
I : Intrinsic plus hand
J : Intrinsic minus hand
1 : Rigid extension of the distal interphalangeal joint when the patient is asked to
extend the middle phalanx against resistance
Correct answer:
D
Your answer:
D
2 : Less interphalangeal flexion when the metacarpo-phalangeal joints are hyper
extended as compared to when the metacarpo-phalangeal joints are flexed
Correct answer:
I
Your answer:
I
3 : Metacarpal grind test
Correct answer:
F
Your answer:
F

References
Green's Operative Hand Surgery

Question 142
With congenital constriction ring syndrome in the hand:
A : Simple constriction rings
B : Rings + distal deformity
C : Rings + distal fusion
D : Amputations

E : Urgent surgical treatment


F : Delayed surgical treatment
G : Staged surgical treatment
H : Z-plasty
1 : Acrosyndactyly implies
Correct answer:
C
Your answer:
A
2 : Tight constrictions on the digits or extremities with vascular compromise
Correct answer:
E
Your answer:
E
3 : When two rings are adjacent the preferred option is
Correct answer:
G
Your answer:
G

References
GREEN'S CHAPTER 40
rare condition, constriction ring syndrome is classified into 4 types: simple constriction rings,
rings+ distal deformity, rings + distal tethering, amputations. acrosyndactyly refers to rings +
distal tethering. early release of the tethering prevents development of deformity but formal
correction is delayed till the child is older.

You might also like